EXAM #2 Practice NCLEX Questions

Pataasin ang iyong marka sa homework at exams ngayon gamit ang Quizwiz!

Error-reduction strategies during medication administration include which of the following? (Select all that apply.) A. quiet zone signs at entrance to the medication room B. protocols and checklist outlining medication administration C. wearing of a sash or vest to signal others to avoid interruptions of the nurse during medication administration D. carrying several patients' prescanned medications on a tray

A, B, and C. All of these strategies, except for carrying several clients' prescanned medications on a cart, potentially reduce the risk of medication error. The work-around of removing more than one client's medication from an automated medication dispensing system could lead to the administration of the wrong medication to a client.

A patient is admitted with a nonhealing surgical wound. Which nursing interventions will the nurse use to promote wound healing? Select all that apply. A. Applying sterile dressing supplies B. Discussing zinc supplementation with the health care provider C. Maintaining bedrest D. Performing careful hand hygiene E. Teaching the patient to increase protein in the diet F. Suggesting the patient consume vitamin C-containing foods.

A. Applying sterile dressing supplies B. Discussing zinc supplementation with the health care provider D. Performing careful hand hygiene E. Teaching the patient to increase protein in the diet F. Suggesting the patient consume vitamin C-containing foods. a, b, d, e, f. Careful hand washing (medical asepsis) is the most important. The nurse will use sterile dressings and supplies and promote intake of vitamins, zinc, and protein. Depending on the site of the wound and condition of the patient, bedrest may be indicated.

The nurse has administered lacosamide to the wrong patient. What is the first action the nurse should take? A. Assess the patient's vital signs and level of consciousness. B. Notify the physician. C. Fill out an incident report. D. Call the respiratory therapist for administration of oxygen.

A. Assess the patient's vital signs and level of consciousness. A. The nurse first assesses the client's vital signs and level of consciousness. Then, it is necessary to notify the physician, implement orders per the physician's direction, and fill out the incident report.

A nurse is teaching parents in a parenting class about the use of car seats and restraints for infants and children. What should be the focus of this education? A. Booster seats should be used for children until they are 4′9″ tall and weigh between 80 and 100 lb. B. Most U.S. states mandate the use of infant car seats and carriers when transporting a child in a motor vehicle. C. Infants and toddlers up to age 2 years (or the maximum height and weight for the seat) should be in a front-facing safety seat. D. Children age older than 6 years may be restrained using a car seat belt in the back seat.

A. Booster seats should be used for children until they are 4′9″ tall and weigh between 80 and 100 lb. a. Booster seats should be used for children until they are 4′9″ tall and weigh between 80 and 100 lb. All 50 U.S. states mandate the use of infant car seats and carriers when transporting a child in a motor vehicle. Infants and toddlers up to age 2 years (or up to the maximum height and weight for the seat) should be in a rear-facing safety seat. Many children older than age 6 years should still be in a booster seat.

The nurse is assessing the pain of a neonate who is admitted to the NICU with a heart defect. Which pain assessment scale would be the best tool to use with this patient? A. CRIES B. COMFORT C. FLACC D. FACES

A. CRIES a. The CRIES Pain Scale is a tool intended for use with neonates and infants from 0 to 6 months. The COMFORT Scale, used to assess pain and distress in critically ill pediatric patients, relies on six behavioral and two physiologic factors that determine the level of analgesia needed to adequately relieve pain in these children. The FLACC Scale (F—Faces, L—Legs, A—Activity, C—Cry, C—Consolability) was designed for infants and children from age 2 months to 7 years who are unable to validate the presence or severity of pain. The FACES Scale is used for children who can compare their pain to the faces depicted on the scale.

A prescriber has written an order for levothyroxine sodium 50 mg/day by mouth. The nurse knows that the standard dose is 50 mcg. What action should the nurse take? A. Call the prescriber and question the order. B. Administer 50 mcg instead. C. Consult the pharmacist about the order. D. Ask the patient what he or she usually takes.

A. Call the prescriber and question the order. A. In the event, a medication is ordered with a potentially wrong abbreviation, the nurse should call the prescriber and question the dosage. Also, it is essential to write out micrograms or milligrams to prevent error.

A nurse working in a long-term care facility uses proper principles of ergonomics when moving and transferring patients to avoid back injury. Which action should be the focus of these preventive measures? A. Carefully assessing the patient care environment B. Using two nurses to lift a patient who cannot assist C. Wearing a back belt to perform routine duties D. Properly documenting the patient lift

A. Carefully assessing the patient care environment a. Preventive measures should focus on careful assessment of the patient care environment so that patients can be moved safely and effectively. Using lifting teams and assistive patient handling equipment rather than two nurses to lift increases safety. The use of a back belt does not prevent back injury. The methods used for safe patient handling and mobility should be documented but are not the primary focus of interventions related to injury prevention.

After instituting a new system for recording patient data, a nurse evaluates the "usability" of the system. Which actions will the nurse include in this step? Select all that apply. A. Checking that the screens are formatted to allow for ease of data entry B. Reordering the screen sequencing to maximize effective use of the system C. Ensuring that the computers can be used by specified users effectively D. Checking that the system is intuitive and supportive of nurses E. Improving end-user skills and satisfaction with the new system F. Ensuring patient data is shareable across health care systems

A. Checking that the screens are formatted to allow for ease of data entry C. Ensuring that the computers can be used by specified users effectively D. Checking that the system is intuitive and supportive of nurses Usability refers to the extent to which a product can be used by specified users to achieve specified goals with effectiveness, efficiency, and satisfaction in a specified context of use. Checking that screens are formatted to allow ease of data entry, ensuring that computers can be used by specified users effectively, and checking that the system is intuitive and supportive of nurses are all tasks related to the "usability" of the system. Maximizing screen sequencing and improving end-user skills and satisfaction with the new system refer to optimization. The ability to share patient data across health care systems is termed interoperability.

The nurse is caring for a patient 1 day postoperative abdominal surgery. The nurse identifies the patient is at risk for wound dehiscence. What patient risk factor is consistent with development of this problem? A. Cigar smoker B. Wound drainage 120 mL over 24 hours C. Height, 5′ 6″ and weight 240 lb D. WBC count 9,500 c/mm3

A. Cigar smoker a. Wound dehiscence is the partial or total separation of wound layers as a result of excessive stress on unhealed wounds. Patients at greater risk include obese or malnourished individuals; tobacco smokers; and those taking anticoagulants, who have infected wounds, or who experience excessive coughing, vomiting, or straining (Hinkle & Cheever, 2018). An increase in the flow of (serosanguineous) fluid from the wound between postoperative days 4 and 5 may be a sign of an impending dehiscence. The patient may say that "something has suddenly given way."

A nurse is using the Katz Index of Independence in Activities of Daily Living (ADLs) to assess the mobility of a hospitalized patient. During the patient interview, the nurse documents the following patient data: "Patient bathes self completely but needs help with dressing. Patient toilets independently and is continent. Patient needs help moving from bed to chair. Patient follows directions and can feed self." Based on this data, which score would the patient receive on the Katz index? 2 4 5 6

4 The total score for this patient is 4. On the Katz Index of Independence in ADLs, one point is awarded for independence in each of the following activities: bathing, dressing, toileting, transferring, continence, and feeding.

Which assessments and interventions should the nurse consider when performing tracheal suctioning? Select all that apply. A. Closely assessing the patient before, during, and after the procedure B. Hyperoxygenating the patient before and after suctioning C. Limiting the application of suction to 20 to 30 seconds D. Monitoring the pulse to detect effects of hypoxia and stimulation of the vagus nerve E. Using an appropriate suction pressure (80 to 150 mm Hg) F. Inserting the suction catheter no further than 1 cm past the length of the tracheal or endotracheal tube

A. Closely assessing the patient before, during, and after the procedure B. Hyperoxygenating the patient before and after suctioning D. Monitoring the pulse to detect effects of hypoxia and stimulation of the vagus nerve E. Using an appropriate suction pressure (80 to 150 mm Hg) F. Inserting the suction catheter no further than 1 cm past the length of the tracheal or endotracheal tube a, b, d, e, f. Close assessment of the patient before, during, and after the procedure is necessary to identify complications such as hypoxia, infection, tracheal tissue damage, dysrhythmias, and atelectasis. The nurse should hyperoxygenate the patient before and after suctioning and limit the application of suction to 10 to 20 seconds. In addition, monitor the patient's pulse frequently to detect potential effects of hypoxia and stimulation of the vagus nerve. Using an appropriate suction pressure (80 to 150 mm Hg) will help prevent atelectasis caused by excessive negative pressure. Research suggests that insertion of the suction catheter should be limited to a predetermined length (no further than 1 cm past the length of the tracheal or endotracheal tube) to avoid tracheal mucosal damage.

When assessing the skin, nurses use techniques to provide complete data and correct documentation. Which actions are appropriate during the skin assessment? Select all that apply. A. Comparing bilateral parts for symmetry B. Proceeding in a toe-to-head, systematic manner C. Using standard terminology to communicate and document findings D. Avoiding using data from the nursing history to direct the assessment E. Documenting only skin abnormalities on the health record F. When risk factors are identified, following up with a related skin assessment

A. Comparing bilateral parts for symmetry B. Proceeding in a toe-to-head, systematic manner C. Using standard terminology to communicate and document findings F. When risk factors are identified, following up with a related skin assessment a, b, c, f. During skin assessment, the nurse should compare bilateral parts for symmetry, use standard terminology to communicate and document findings, and perform the appropriate skin assessment when risk factors are identified. The nurse should proceed in a head-to-toe systematic manner, using cues/data from the nursing history to direct the assessment. When documenting a physical assessment of the skin, the nurse should describe exactly what is observed or palpated, including appearance, texture, size, location or distribution, and characteristics of any findings.

A nurse is caring for patients who are nonverbal. What are examples of behavioral responses to pain? Select all that apply. A. Cradling a wrist that was injured in a car accident B. Moaning and crying from abdominal pain C. Increasing pulse following a myocardial infarction D. Striking out at a nurse who attempts to provide a bath E. Acting depressed and withdrawn while experiencing chronic cancer pain F. Pulling away from a nurse trying to give an injection

A. Cradling a wrist that was injured in a car accident B. Moaning and crying from abdominal pain F. Pulling away from a nurse trying to give an injection a, b, f. Physiologic responses are involuntary body responses; behavioral responses reflect body movements; affective responses reflect mood and emotions. Protecting or guarding a painful area, moaning and crying, and moving away from painful stimuli are behavioral responses. Examples of a physiologic or involuntary response would be increased blood pressure or dilation of the pupils. Affective responses, such as anger, withdrawal, and depression, are psychological in nature.

A nurse is using the steps in informatics evaluation to assess a patient portal. Which activities might occur in the "determining the question" step? Select all that apply. A. Developing a clear, focused question to determine the data to be collected B. Determining what to evaluate C. Determining how the data ultimately should be reported D. Deciding what specific data elements need to be collected E. Clarifying exactly how the data will be collected F. Performing comprehensive documentation of the data collected

A. Developing a clear, focused question to determine the data to be collected C. Determining how the data ultimately should be reported a, c. The nurse develops a clear, focused question to determine the data to be collected and the nurse determines how the data ultimately should be reported during the "determine the question" step. The nurse determines what to evaluate during the step "determine what will be evaluated." The nurse decides what specific data elements need to be collected during the "determine the needed data" step. The nurse clarifies exactly how the data will be collected during the "determine the data collection method and sample size" step. The nurse performs comprehensive documentation of the data collected during the "document your outcome evaluation" step.

Thirty-six hours after having surgery, a patient has a slightly elevated body temperature and generalized malaise as well as pain with redness at the surgical site. Which action is most appropriate? MAR - Acetaminophen 650 mg every 6 hours prn fever - Cefazolin (antibiotic) 1 g 1 hour preoperatively - Cefazolin 1 g, every 6 hours 3 times, postoperatively A. Documenting the findings and continuing to monitor the patient B. Administering antipyretics and contacting the provider for an antibiotic prescription C. Increasing the frequency of assessment to every hour and notifying the patient's primary care provider D. Obtaining a wound culture and increasing the frequency of wound care

A. Documenting the findings and continuing to monitor the patient a. The assessment findings are normal for this stage of healing following surgery. The patient is in the inflammatory phase of the healing process, which involves a response by the immune system. This acute inflammation is characterized by pain, heat, redness, and swelling at the site of the injury (surgery, in this case). The patient also has a generalized body response, including a mildly elevated temperature, leukocytosis, and generalized malaise.

A nurse is teaching a nursing student how to perform perineal care for patients. What actions are appropriate when performing this procedure? Select all that apply. A. For male and female patients, wash the groin area with a small amount of soap and water and rinse. B. For a female patient, spread the labia and move the washcloth from the anal area toward the pubic area. C. For male and female patients, always proceed from the most contaminated area to the least contaminated area. D. For male and female patients, use a clean portion of the washcloth for each stroke. E. For a male patient, clean the tip of the penis first, moving the washcloth in a circular motion from the meatus outward. F. In an uncircumcised male patient, avoid retracting the foreskin (prepuce) while washing the penis.

A. For male and female patients, wash the groin area with a small amount of soap and water and rinse. D. For male and female patients, use a clean portion of the washcloth for each stroke. E. For a male patient, clean the tip of the penis first, moving the washcloth in a circular motion from the meatus outward. a, d, e. Wash and rinse the groin area (both male and female patients) with a small amount of soap and water, and rinse. For male and female patients, always proceed from the least contaminated area to the most contaminated area and use a clean portion of the washcloth for each stroke. For a male patient, clean the tip of the penis first, moving the washcloth in a circular motion from the meatus outward. For a female patient, spread the labia and move the washcloth from the pubic area toward the anal area. In an uncircumcised male patient (teenage or older), retract the foreskin (prepuce) while washing the penis and return it to its original position when finished.

Nursing students enrolled in an informatics course question the purpose of "big data." Which statement do the students find best describes use of big data? A. Gathering data from multiple sources to help organizations answer questions and make predictions in multiple settings B. Predicting patients likely to be readmitted, allowing early intervention C. Promoting the most efficient staffing model for the organization D. Developing charts or graphs to visualize large amounts of complex data

A. Gathering data from multiple sources to help organizations answer questions and make predictions in multiple settings a. As organizations transition from traditional fee-for-service models to value-based payment models, information about populations rather than individuals is needed. Big data comprises the accumulation of health care-related data from multiple sources, combined with new technologies that allow for the transformation of data to information for patient care regardless of setting. Sources of available data can include the EHR; medical devices such as monitors, ventilators, and smart pumps; radiology, laboratory, and pathology systems; wearable devices and home monitoring systems; financial databases; genomics; open sources; patient portals; and real-time location systems. Predictive analytics uses statistical techniques to make predictions about future or unknown events, such as patients who are at risk for readmission. Data visualization presents data in a pictorial or graphical format for analysis. The growing population of older adults with complex health needs and a potential understaffing are important but are not the driving force for the development of this technology.

A nurse on a surgical unit is working with a nursing student and discussing various phases of wound healing for postoperative patients. Which statements accurately describe these stages? Select all that apply. A. Hemostasis occurs immediately after the initial injury. B. A liquid called exudate is formed during the proliferation phase. C. White blood cells move to the wound in the inflammatory phase. D. Granulation tissue forms in the inflammatory phase. E. During the inflammatory phase, the patient has generalized body response. F. A scar forms during the proliferation phase.

A. Hemostasis occurs immediately after the initial injury. C. White blood cells move to the wound in the inflammatory phase. E. During the inflammatory phase, the patient has generalized body response. a, c, e. Hemostasis occurs immediately after the initial injury, and exudate occurs in this phase as plasma and blood components leak out into the injured area. White blood cells, predominantly leukocytes and macrophages, move to the wound in the inflammatory phase to ingest bacteria and cellular debris. During the inflammatory phase, the patient has a generalized body response, including a mildly elevated temperature, leukocytosis (increased number of white blood cells in the blood), and generalized malaise. New tissue, called granulation tissue, forms the foundation for scar tissue development in the proliferation phase. New collagen continues to be deposited in the maturation phase, which forms a scar.

A nurse is caring for a patient with chronic lung disease who is receiving oxygen through a nasal cannula. What nursing action is performed correctly? Select all that apply. A. Making sure the oxygen is flowing into the prongs B. Maintaining oxygen saturation between 94% and 98% C. Encouraging the patient to breathe through their nose with their mouth closed D. Initiating the oxygen flow rate at 6 L/min or more E. Protecting the patient's skin from irritation by the oxygen tubing

A. Making sure the oxygen is flowing into the prongs C. Encouraging the patient to breathe through their nose with their mouth closed E. Protecting the patient's skin from irritation by the oxygen tubing a, c, e. The nurse should assure that the oxygen is flowing out of the prongs prior to inserting them into the patient's nostrils. The nurse should encourage the patient to breathe through their nose with the mouth closed. The nurse should adjust the flow rate and maintain the patient's oxygen saturation as prescribed. The nurse should implement pressure injury prevention strategies; pressure from the tubing could result in medical device-related alterations in skin integrity.

A nurse caring for a patient with a stage 3 pressure wound with tunneling. How will the nurse best assess the tunneled area? A. Moisten a sterile, flexible applicator with saline and insert it gently into the wound at a 90-degree angle with the tip down. B. Photograph the wound per policy and describe the estimated depth in centimeters. C. Gently insert a sterile applicator into the wound and move it in a clockwise direction. D. Insert a calibrated probe gently into the wound and mark the point that is even with the surrounding skin surface.

A. Moisten a sterile, flexible applicator with saline and insert it gently into the wound at a 90-degree angle with the tip down. a. To measure the depth of a wound, the nurse should perform hand hygiene and apply gloves; moisten a sterile, flexible applicator with saline and insert it gently into the wound at a 90-degree angle with the tip down; mark the point on the swab that is even with the surrounding skin surface, or grasp the applicator with the thumb and forefinger at the point corresponding to the wound's margin; and remove the swab and measure the depth with a ruler.

A nurse is caring for patients in a hospital setting. Which patient would the nurse place at risk for pain related to the mechanical activation of pain receptors? A. Older adult on bedrest following cervical spine surgery B. Patient with a severe sunburn being treated for dehydration C. Industrial worker who has burns caused by a caustic acid D. Patient experiencing cardiac disturbances from an electrical shock

A. Older adult on bedrest following cervical spine surgery a. Receptors in the skin and superficial organs may be stimulated by mechanical, thermal, chemical, and electrical agents. Friction from bed linens causing pressure sores is a mechanical stimulant. Sunburn is a thermal stimulant. An acid burn is the result of a chemical stimulant. An electrical shock is an electrical stimulant.

When signing in to the electronic medical record, a nurse is prompted to change their password. Which best-practice guideline does the nurse use to formulate the new password? A. Password should be at least 8 characters long and unique. B. Passwords should be memorable and used in other applications. C. Date of birth makes an ideal password. D. Nurses should share their passwords with their nurse managers.

A. Password should be at least 8 characters long and unique. a. Passwords should contain 8 to 15 characters, never be shared or re-used, and be easy to remember but hard to guess. Using multifactor authentication and a password manager can help organize and protect information. The password should not be shared with the nurse manager or others.

A nurse in a rehabilitation facility is evaluating patients with chronic pain to develop an interprofessional plan of care. Which patients would the nurse identify who could benefit from a multimodal approach to pain management? Select all that apply. A. Patient receiving chemotherapy for bladder cancer B. Adolescent who had an appendectomy C. Patient who is experiencing a ruptured aneurysm D. Patient with fibromyalgia requesting pain medication E. Patient having back pain related to an accident that occurred last year F. Patient experiencing pain from second-degree burns

A. Patient receiving chemotherapy for bladder cancer D. Patient with fibromyalgia requesting pain medication E. Patient having back pain related to an accident that occurred last year a, d, e. Chronic pain is pain that may be limited, intermittent, or persistent but that lasts beyond the normal healing period. Examples are cancer pain, fibromyalgia pain, and back pain. Acute pain is generally rapid in onset and varies in intensity from mild to severe, as occurs with an emergency appendectomy, a ruptured aneurysm, and pain from burns.

When the nurse assists a patient recovering from abdominal surgery to walk, the nurse observes that the patient grimaces, moves stiffly, and becomes pale. The nurse received in shift report that the patient has consistently refused pain medication. To help promote comfort, which additional data will the nurse gather? Select all that apply. A. Patient's understanding of or fear of taking prescribed analgesics B. Assessment of any current pain C. Presence of anxiety or additional stressors D. Assessment of the surgical incision for infection E. What the patient has eaten to this point F. Whether the patient is using the incentive spirometer

A. Patient's understanding of or fear of taking prescribed analgesics B. Assessment of any current pain C. Presence of anxiety or additional stressors D. Assessment of the surgical incision for infection a, b, c, d. While it seems the patient's immediate problem is unrelieved pain because the patient refuses to take pain medication, through further assessment, the nurse can plan to address fears of medication, teach about use of the pump, determine if anxiety is interfering with pain, or an infection is causing increased pain. While decreased oral intake may be a response to pain, the patient's dietary intake will not uncover the underlying reason for refusing medications. Use of the incentive spirometer is not included in pain assessment; rather, it is an intervention to prevent atelectasis.

A student with a history of asthma visits the school nurse reporting difficulty breathing and wheezing. Which tool would the nurse use to assess the severity of airway resistance? A. Peak flow meter B. End-tidal CO2 monitor C. Chest tube D. Arterial blood gas

A. Peak flow meter a. A peak flow meter is used to assess the point of highest flow during forced expiration. It is routinely used by and for patients with moderate or severe asthma to measure the severity of the disease and degree of disease management. Capnography or end-tidal CO2 monitoring is used for assessing and monitoring ventilation and placement of artificial airways, predicting patients who are at risk for respiratory compromise, are experiencing partial or complete airway obstruction, or are experiencing hypoventilation (Burns & Delgado, 2019; Seckel, 2018). A chest tube is used to remove air or fluid from the pleural space. The arterial blood gas (ABG) is used to assess oxygenation, ventilation, and acid-base status; it is invasive and not performed in the school setting.

A nurse is assessing a patient with COPD who is experiencing dyspnea. What action will the nurse take first? A. Place the patient in Fowler position. B. Encourage diaphragmatic breathing. C. Ask the patient to cough. D. Initiate oral suctioning of secretions.

A. Place the patient in Fowler position. a. Patients with COPD experience dyspnea related to problems with ventilation and/or hypoxemia. One of the most common symptoms of hypoxia is dyspnea (difficulty breathing). Elevating the head of the bed will improve respiratory expansion and oxygenation. Coughing to facilitate secretion removal, pursed-lip breathing, and/or diaphragmatic breathing may be indicated, after sitting the patient up. Suction is indicated for patients demonstrating the presence of secretions, such as adventitious breath sounds or moist cough with phlegm; there is no indication this patient requires suctioning at this time.

A nurse caring for patients in a critical care unit knows that providing good oral hygiene is an essential to good patient outcomes, especially for those receiving mechanical ventilation. What are positive outcomes expected from this care? Select all that apply. A. Promoting the patient's sense of well-being B. Preventing deterioration of the oral cavity C. Contributing to decreased incidence of aspiration pneumonia D. Eliminating the need for flossing E. Decreasing oropharyngeal secretions F. Compensating for an inadequate diet

A. Promoting the patient's sense of well-being B. Preventing deterioration of the oral cavity C. Contributing to decreased incidence of aspiration pneumonia a, b, c. Adequate oral hygiene is essential for promoting the patient's sense of well-being and preventing deterioration of the oral cavity. Diligent oral hygiene and use of chlorhexidine gluconate (CHG) in critical care areas, can limit the growth of pathogens in oropharyngeal secretions, decreasing the incidence of ventilator-associated pneumonia, aspiration pneumonia, and other systemic diseases. Oral care does not eliminate the need for flossing, decrease oropharyngeal secretions, or compensate for poor nutrition.

An experienced nurse and new graduate nurse are caring for a confused older adult who gets out of bed and wanders. The preceptor intervenes when observing which action by the graduate nurse? A. Raising all four side rails to keep the patient in bed B. Performing documentation in the patient's room C. Suggesting obtaining a patient "sitter" D. Using a bed alarm to alert staff the patient leaving the bed

A. Raising all four side rails to keep the patient in bed a. The desire to prevent a patient from wandering is not sufficient reason for the use of side rails. People of small stature are more likely to be injured slipping through or between the side rails. A history of falls from a bed with raised side rails carries a significant risk for a serious incident. The nurse uses creative measures while promoting safety and respect for the patient's dignity.

The hospital's fire alarm sounds, and an announcement is made that there is a fire in a patient room. What is the priority for nurses on the unit? A. Removing patients from the room or vicinity B. Attempting to put out the fire with water or appropriate extinguishers C. Closing all the doors on the unit to contain the fire D. Running to the closest unit and requesting help

A. Removing patients from the room or vicinity a. The nurse uses the acronym RACE and rescues and/or removes the patient and those in nearby rooms as the safety priority. Sounding the alarm and extinguishing the fire are important after the patient is safe. Remaining on the unit allows you to assist patients and is more appropriate; assistance can be summoned by phone.

A nurse in the PACU is performing oral suctioning for a patient with an oropharyngeal airway, when the patient begins to vomit. What is the nurse's priority nursing action at this time? A. Removing the suction catheter and elevating the head of the bed B. Notifying the primary health care provider C. Confirming the size of the oral airway is correct D. Placing the patient in the supine position

A. Removing the suction catheter and elevating the head of the bed a. The nurse discontinues suctioning, elevates the head of the bed, and turns the patients to the side to prevent aspiration. Airway protection takes priority; after positioning the patient, the nurse continues to suction the airway and oropharynx. Once airway patency has been established, the nurse will notify the provider of vomiting. There is no indication the oral airway is too large. Placing the patient supine while vomiting is inappropriate, as that could promote aspiration.

A nurse incorporates use of the patient portal in the discharge teaching for a patient who received a kidney transplant. What services does the nurse explain are available with this technology? Select all that apply. A. Reviewing diagnostic and lab test results B. Making follow-up appointments C. Using health and fitness apps D. Entering clinical data, such as blood pressure E. Providing digital medication reminders F. Requesting prescription refills

A. Reviewing diagnostic and lab test results B. Making follow-up appointments C. Using health and fitness apps D. Entering clinical data, such as blood pressure F. Requesting prescription refills The patient portal can streamline services by allowing access to medical history and other health information; facilitating completion of online forms and questionnaires; offering secure and convenient communication with providers; and having the ability to place request prescription refills, pay bills, review lab results, schedule appointments, receive reminders for screenings and immunizations, enter clinical data such as blood pressure, glucose levels, weight, and other activity tracking data, review progress notes, and access educational materials based on diagnosis or procedure.

A nurse is providing active-assistive range-of-motion exercises for a patient who is recovering from a stroke. During the session, the patient reports that they are "too tired to go on." What actions are appropriate at this time? Select all that apply. A. Stop performing the exercises. B. Decrease the number of repetitions performed. C. Reevaluate the plan of care. D. Move to the patient's other side to perform exercises. E. Encourage the patient to finish the exercises and then rest. F. Assess the patient for additional symptoms of intolerance.

A. Stop performing the exercises. C. Reevaluate the plan of care. F. Assess the patient for additional symptoms of intolerance. a, c, f. When a patient reports fatigue during range-of-motion exercises, the nurse should stop the activity, reevaluate the plan of care, and assess the patient for further symptoms indicating the activity is not tolerated. The exercises can be rescheduled for times of the day when the patient is feeling more rested, or spaced out at different times of the day.

A nurse works in a facility stating they support a culture of safety. What will the nurse expect to find operationalized in this culture? Select all that apply. A. Support for reporting errors and near misses without blame B. Nurses being the employees responsible for safety in the organization C. Commitment of resources to address actual/potential safety issues D. Emphasis placed on individuals, their departments, and resources E. Promotion of teamwork and collaboration throughout the organization F. Administrators' and managers' commitment to safe operations

A. Support for reporting errors and near misses without blame C. Commitment of resources to address actual/potential safety issues E. Promotion of teamwork and collaboration throughout the organization F. Administrators' and managers' commitment to safe operations a, c, e, f. The key features of a culture of safety include: (1) acknowledging the high-risk nature of health care and the commitment to safe operations, (2) maintaining a blame-free environment where reporting is protected and expected, (3) promoting teamwork and collaboration to prevent and seek solutions to patient safety issues, and (4) valuing safety as a focus in all health care facilities, the home, workplace, and community.

A nurse is instructing a patient recovering from a stroke on proper use of a cane. What information will the nurse include in the teaching plan? A. Support weight on the stronger leg and cane and advance weaker foot forward. B. Hold the cane in the same hand of the leg with the most severe deficit. C. Stand with as much weight distributed on the cane as possible. D. Avoid using the cane to rise from a sitting position, as this is unsafe.

A. Support weight on the stronger leg and cane and advance weaker foot forward. a. The proper procedure for using a cane is to (1) stand with weight distributed evenly between the feet and cane; (2) support weight on the stronger leg and the cane and advance the weaker foot forward, parallel with the cane; (3) support weight on the weaker leg and cane and advance the stronger leg forward ahead of the cane; (4) move the weaker leg forward until even with the stronger leg and advance the cane again as in step 2. The patient should keep the cane within easy reach and use it for support to rise safely from a sitting position.

A nurse is developing an exercise program for a patient who has COPD. Which instructions would the nurse include in a teaching plan for this patient? Select all that apply. A. Teach the patient to avoid sudden position changes that may cause dizziness. B. Recommend that the patient restrict fluid intake until after exercise. C. Instruct the patient to push a little further beyond fatigue each session. D. Tell the patient to avoid exercising in very cold or very hot temperatures. E. Encour age the patient to modify exercise if weak or ill. F. Recommend that the patient consume a high-carb, low-protein diet.

A. Teach the patient to avoid sudden position changes that may cause dizziness. D. Tell the patient to avoid exercising in very cold or very hot temperatures. a, d. Teaching points for exercising for a patient with COPD include avoiding sudden position changes that may cause dizziness and avoiding extreme temperatures. The nurse should also instruct the patient to remain adequately hydrated, respect fatigue as a sign of activity intolerance and not push to the point of exhaustion, and avoid exercise if weak or ill. Older adults should consume a high-protein, high-calcium, and vitamin D-enriched diet.

A nursing student asks an experienced nurse why they provide massage for their patients. Which of these would be reflected in the nurse's response? A. To help with pain management B. To provide comfort C. To communicate to patients through touch D. To energize patients, especially those with dementia E. To facilitate healing after back or spinal surgery F. To help increase circulation

A. To help with pain management B. To provide comfort C. To communicate to patients through touch F. To help increase circulation a, b, c, f. The benefits of massage include general relaxation and increased circulation, pain relief, sleep promotion, and increased patient comfort and well-being. Massage also provides an opportunity for the nurse to communicate and connect with the patient through touch. Back massage is contraindicated if the patient has had back surgery or has fractured ribs.

A nurse is caring for a patient with lower extremity paralysis. Which action will the nurse take to prevent external rotation of the hip and foot? A. Use a trochanter roll. B. Apply SCDs. C. Obtain a prescription for antiembolism stockings. D. Have the patient maintain low-Fowler's position. E. Have the patient cross their arms on their chest and place a pillow between their knees. F. Place a cervical collar on the patient's neck and gently roll them to the other side of the bed.

A. Use a trochanter roll. a. The trochanter roll is used to support the hips and legs to prevent external rotation. SCDs and antiembolism stockings are used to prevent DVT. Fowler's position allows for foot rotation and increases sacral pressure.

The nurse is cleaning an open abdominal wound that has edges that are not approximated. What are accurate steps in this procedure? Select all that apply. A. Use standard precautions or transmission-based precautions when indicated. B. Moisten a sterile gauze pad or swab with the prescribed cleansing agent and squeeze out excess solution. C. Clean the wound in full or half circles beginning on the outside and working toward the center. D. Work outward from the incision in lines that are parallel to it from the dirty area to the clean area. E. Clean to at least 1 inch beyond the end of the new dressing if one is being applied. F. Clean to at least 3 inches beyond the wound if a new dressing is not being applied.

A. Use standard precautions or transmission-based precautions when indicated. B. Moisten a sterile gauze pad or swab with the prescribed cleansing agent and squeeze out excess solution. E. Clean to at least 1 inch beyond the end of the new dressing if one is being applied. a, b, e. The correct procedure for cleaning an open wound with edges that are not approximated is: (1) use standard precautions and appropriate transmission-based precautions when indicated, (2) moisten sterile gauze pad or swab with prescribed cleansing agent and squeeze out excess solution, (3) use a new swab or gauze for each circle, (4) clean the wound in full or half circles beginning in the center and working toward the outside, (5) clean to at least 1 inch beyond the end of the new dressing, and (6) clean to at least 2 inches beyond the wound margins if a dressing is not being applied.

A nurse is caring for an adolescent with severe acne. Which recommendations would be most appropriate to include in the teaching plan for this patient? Select all that apply. A. Wash the skin twice a day with a mild cleanser and warm water. B. Use cosmetics liberally to cover blackheads. C. Apply emollients on the area. D. Squeeze blackheads as they appear. E. Keep hair off the face and wash hair daily. F. Avoid tanning booth exposure and use sunscreen.

A. Wash the skin twice a day with a mild cleanser and warm water. E. Keep hair off the face and wash hair daily. F. Avoid tanning booth exposure and use sunscreen. a, e, f. Washing the skin removes oil and debris, hair should be kept off the face and washed daily to keep oil from the hair off the face. Exposure to UV light should be avoided, especially when using acne treatments. Liberal use of cosmetics and emollients can clog the pores, worsening acne. Squeezing blackheads is discouraged because it may lead to infection.

During an initial nursing assessment, the patient reports that he is allergic to a particular medicine. What should the nurse ask the patient? A. What symptoms occurred when you had the allergic reaction? B. Did you need to take epinephrine (Adrenalin)? C. Did your physician think this information needed to be communicated? D. Have you ever overdosed on this medication?

A. What symptoms occurred when you had the allergic reaction? A. The nurse should inquire what symptoms occurred that prompted the client to say he was allergic to the medication. Sometimes clients state that they are allergic to a drug when they actually have had an adverse reaction such as nausea. If clients are truly allergic, resulting in an anaphylactic reaction, then it is imperative they not receive the medication.

A nurse on an adult surgical floor enters a patient room and observes a family member pressing the button to administer a dose of PCA via the infusion pump. What response by the nurse is most appropriate? A. "That dose will sure be helpful after their type of surgery." B. "Having only the patient use the pump prevents respiratory complications." C. "If the patient asked you to press the button, then it's OK." D. "Since the pump has built in safeguards, you can help with pain management."`

B. "Having only the patient use the pump prevents respiratory complications." b. Unauthorized family members or caregivers (instead of the patient) who administer PCA by pushing the dosage button can cause serious analgesic overdoses resulting in oversedation, respiratory depression, and death. This is known as PCA by proxy. Institutions should have protocols in place to protect against unauthorized PCA delivery and clear warning labels attached to PCA pumps stating, "WARNING: BUTTON TO BE PRESSED ONLY BY THE PATIENT" (ISMP).

During post-conference, nursing students explore definitions of pain and its nature. Which statements should be included in this discussion? Select all that apply. A. "It is whatever the health care provider treating the pain says it is." B. "Pain exists whenever the person experiencing it says it is present." C. "It is an emotional and sensory reaction to tissue damage." D. "Pain is a simple, universal, and easy-to-describe phenomenon." E. "When a cause cannot be identified, pain is psychological in nature." F. "It is classified by duration, location, source, transmission, and etiology."

B. "Pain exists whenever the person experiencing it says it is present." C. "It is an emotional and sensory reaction to tissue damage." F. "It is classified by duration, location, source, transmission, and etiology." b, c, f. Nurses must respect patients' reports of pain and consider the patient an expert on their pain experience. An accepted definition of pain is that pain is whatever the patient says it is, existing whenever the person says it does, even if the cause is not clearly established." Pain is a complex, unpleasant sensory and emotional experience associated with actual or potential tissue damage. Pain may be classified according to its duration, its location or source, its mode of transmission, or its etiology.

A nurse is providing teaching for a patient who will undergo cardiac surgery and return to the intensive care unit with an endotracheal tube. What education is most important for the nurse to provide? A. "The endotracheal tube will drain out excess secretions from the surgical site." B. "This tube is used to facilitate breathing; you will not be able to speak while it is in place." C. "This is a surgically placed tube in your neck; we will suction it frequently to remove mucus." D. "Your oxygenation will be monitored frequently using pulse oximetry."

B. "This tube is used to facilitate breathing; you will not be able to speak while it is in place." b. Patients with an endotracheal tube are unable to speak. Explaining this to the patient preoperatively, along with information that they will be closely monitored, can help decrease anxiety. The endotracheal tube is used during anesthesia or for mechanical ventilation; it is not a surgical drain. A tracheostomy, located in the neck area, is a surgically placed artificial airway. While pulse oximetry will be used to monitor oxygenation, to prevent undue anxiety, it is most important that the patient understands speech will not be possible.

The patient receives regular insulin 5 units subcutaneously. To what degree is the syringe held for the injection? (Select all that apply.) A. 30 degrees B. 45 degrees C. 60 degrees D. 90 degrees

B. 45 degrees D. 90 degrees B and D. Subcutaneous injections of insulin are administered at 45 or 90 degrees with a 25-gauge needle.

A nurse caring for patients in a pediatric office assesses children's achievement of developmental milestones. Which patient finding requires follow-up with the pediatrician? A. 4-month-old infant who is unable to roll over B. 6-month-old infant who is unable to hold head up C. 11-month-old infant who cannot walk unassisted D. 18-month-old toddler who cannot jump

B. 6-month-old infant who is unable to hold head up b. By 5 months, head control is usually achieved. An infant usually rolls over by 6 to 9 months. By 15 months, most toddlers can walk unassisted. By 2 years, most toddlers can jump.

The nurse is taking care of a man who is confused about the different medications he is prescribed. He notes that some of the drug names have changed over the course of time he has been taking them. When counseling him, it is most important to keep the following statement in mind: A. A drug can belong to only one group or classification. B. A prototype drug is the standard by which similar drugs are compared. C. Drug groups and prototypes change frequently, and knowledge about a prototype cannot guide knowledge about other drugs in the same class. D. The generic name of a drug changes among manufacturers.

B. A prototype drug is the standard by which similar drugs are compared. The first drug to be developed in a group is usually considered the prototype or main example of the group, and similar drugs that are developed later are compared to the prototype. Knowledge of the prototype drug can help the nurse understand the actions of other drugs in the class. Most drugs can belong to multiple groups (e.g., a therapeutic group, a chemical group), depending on various characteristics. Drug groups and prototypes are usually quite stable, and most new drugs fit into a known classification. The trade or brand name changes among manufacturers, but the generic name stays the same.

A nurse is assessing a patient receiving a continuous opioid infusion. For which outcome of treatment would the nurse immediately notify the primary care provider? A. A respiratory rate of 11/min with normal depth B. A sedation level of 4 C. Mild forgetfulness D. Reported constipation

B. A sedation level of 4 b. Sedation levels predict respiratory depression. The sedation scale uses: S = sleep, easy to arouse: no action necessary; 1 = awake and alert; no action necessary; 2 = occasionally drowsy but easy to arouse; requires no action; 3 = frequently drowsy and drifts off to sleep during conversation; decrease the opioid dose; and 4 = somnolent with minimal or no response to stimuli; discontinue the opioid and consider use of naloxone. A respiratory level of 11 with normal depth of breathing is usually not a cause for alarm. Mild forgetfulness or confusion may result from opioids; additional observation is necessary. Constipation is not life threatening; it should be reported to the health care provider but is not the priority.

The nurse applies the gate control theory of pain to provide pain relief to a patient with chronic lower back pain. What nursing intervention will help relieve pain by "closing the gate"? A. Encouraging regular use of analgesics B. Applying moist heat to the area at intervals C. Reviewing the pain experience with the patient D. Ambulating the patient after administering medication

B. Applying moist heat to the area at intervals b. The gate control theory states that a limited amount of sensory information can be processed by the nervous system at any given moment. When too much information is sent through, certain cells in the spinal column interrupt the signal as if closing a gate, interfering with pain perception. Nursing measures such as applying warmth to the lower back stimulate the large nerve fibers to close the gate and block the pain. The other choices do not involve attempts to stimulate large nerve fibers that interfere with pain transmission as explained by the gate control theory.

A nurse is caring for a patient who is on bedrest following a spinal injury. Which action is appropriate to prevent foot drop? A. Maintain the supine position with supination on the feet. B. Ask the family to bring in high-top sneakers to maintain foot dorsiflexion. C. Encourage hyperextension of the feet with adaptive devices or splints. D. Use pillows to keep the feet in the abducted position.

B. Ask the family to bring in high-top sneakers to maintain foot dorsiflexion. b. To prevent foot drop, the nurse should support the feet in dorsiflexion using a footboard and/or high-top sneakers for further support. Supination involves lying patients on their back or facing a body part upward, and hyperextension is a state of exaggerated extension. Abduction involves lateral movement of a body part away from the midline of the body. These positions do not prevent foot drop.

A nurse is caring for a patient admitted for an acute asthma exacerbation. The patient reports extreme dyspnea, stating, "Turn up the oxygen, I'm not getting enough air." Which actions would the nurse take first? A. Suction the airway. B. Assess the pulse oximetry reading. C. Obtain a peak flow meter reading. D. Assess for cyanosis of the lips.

B. Assess the pulse oximetry reading. b. Using the nursing process, the nurse first assesses the oxygen saturation via pulse oximetry before changing the oxygen flow rate. Suctioning is provided to remove respiratory secretions; the nurse would note adventitious breath sounds or phlegm with cough indicating a need for suction. A peak flow meter is used to assess the point of highest flow during forced expiration. It is routinely used for patients with moderate or severe asthma to measure the severity of the disease and degree of disease management. While cyanosis of the lips is a late sign of hypoxemia, the nurse can quickly begin to alleviate or lessen dyspnea by simply repositioning the patient. d. To prevent fatigue during activities including hygiene, the nurse should group (personal care) activities into smaller steps and encourage rest periods between activities. The nurse promotes and maintains dignity, independence, and strength by assisting with activities when the patient has difficulty. The nurse should encourage the patient to voice feelings and concerns about self-care deficits and teach the patient to coordinate pursed-lip or diaphragmatic breathing with the activity.

A nurse is assisting a postoperative patient with conditioning exercises to prepare for ambulation. Which instructions from the nurse are appropriate for this patient? Select all that apply. A. Do full-body pushups in bed six to eight times daily. B. Breathe in and out smoothly during quadricep-setting exercises. C. Place the bed in the lowest position or use a footstool for dangling. D. Dangle on the side of the bed for 30 to 60 minutes. E. Allow the nurse to bathe you completely to prevent fatigue. F. Perform quadriceps two to three times per hour, four to six times a day.

B. Breathe in and out smoothly during quadricep-setting exercises. C. Place the bed in the lowest position or use a footstool for dangling. F. Perform quadriceps two to three times per hour, four to six times a day. b, c, f. Breathing in and out smoothly during quadricep-setting exercises maximizes lung inflation. The patient should perform quadricep-setting exercises two to three times per hour, four to six times a day, or as ordered. The patient should never hold their breath during exercise drills because this places a strain on the heart. Pushups are usually done three or four times a day and involve only the upper body. Dangling for a few minutes is done to adjust to the upright position; dangling for 30 to 60 minutes is impractical for the nurse to supervise and may prove unsafe. The nurse should place the bed in the lowest position or use a footstool for dangling. The nurse should also encourage the patient to be as independent as possible to prepare for return to normal ambulation and ADLs.

A nurse informaticist is using the steps of the SDLC to design a new system for home health care documentation. The nurse analyzes the old system and develops plans for the new system. What step of this process will the nurse take next? A. Testing B. Designing C. Implementing D. Evaluating

B. Designing b. The SDLC focuses on the areas of Analyze and Plan, Design and Build, Test, Train, Implement, Maintain, and Evaluate. After analyzing and planning the new system, the nurse would move on to the design step in which the basic design of the new system is developed. The nurse would then test the system, train employees, and implement, maintain, and evaluate the new system in that order.

What should the nurse keep in mind when evaluating a patient's response to drug therapy? A. Few drugs cause adverse effects. B. Drugs may cause virtually any symptom or problem. C. Patients always report adverse effects. D. Therapeutic effects are more important than adverse effects.

B. Drugs may cause virtually any symptom or problem. B. It is important to know that drugs may cause virtually any symptom or problem.

A nurse has exhausted every effort to keep a confused, postoperative patient safe and in bed. Following The Joint Commission guidelines for use of restraints, which nursing action reflects safe practice? A. Positioning the patient in the supine position prior to applying wrist restraints B. Ensuring that two fingers can be inserted between the restraint and patient's wrist C. Applying a cloth restraint to the left hand of the patient with an IV catheter in the right wrist D. Tying an elbow restraint to the raised side rail of the patient's bed

B. Ensuring that two fingers can be inserted between the restraint and patient's wrist b. The nurse should be able to place two fingers between the restraint and a patient's wrist or ankle. Restraining the patient in a supine position increases the risk of aspiration. Due to the IV in the right wrist, alternative forms of restraints should be tried, such as a cloth mitt or an elbow restraint. Securing the restraint to a side rail may injure the patient when the side rail is lowered.

A nurse notes a pressure wound base is red. Using the RYB system for documentation, what intervention is indicated? A. Irrigating the wound and applying an absorbent dressing B. Gently cleansing the wound and applying a moist dressing C. Discussing consultation for surgical debridement with the provider D. Performing frequent dressing changes to keep the wound and dressing dry

B. Gently cleansing the wound and applying a moist dressing b. Red wounds are in the proliferative stage of healing and reflect the color of normal granulation tissue. Wounds in this stage need protection with nursing interventions that include gentle cleansing, use of moist dressings, and dressing changes only when necessary (or based on product manufacturer's recommendations). To cleanse yellow wounds, nursing interventions include the use of wound cleansers and irrigation. The eschar found in black wounds requires debridement (removal) before the wound can heal.

The nurse manager and nurses in an acute care hospital are participating in a safety huddle to identify patients at risk for falling. Which patients will the nurses determine require follow-up? Select all that apply. A. Age >50 years B. History of falling C. Taking antibiotics D. Presence of postural hypotension E. Nausea from chemotherapy F Transferred from long-term care

B. History of falling D. Presence of postural hypotension F Transferred from long-term care b, d, f. Risk factors for falls include age >65 years, documented history of falls, postural hypotension which can cause dizziness, and unfamiliar environment. A medication regimen that includes diuretics creating urinary urgency and tranquilizers, sedatives, hypnotics, or analgesics causing altered mental status and impaired judgment are also risks. Chemotherapy or antibiotics are not included as factors leading to falls.

When assessing pain in a child, the nurse needs to be aware of what considerations? A. Immature neurologic development results in reduced pain sensation B. Inadequate or inconsistent relief of pain is widespread C. Reliable assessment tools are currently unavailable D. Narcotic analgesic use should be avoided

B. Inadequate or inconsistent relief of pain is widespread b. Health care personnel are placing awareness of pain relief in children as a priority. The evidence supports the fact that children do indeed feel pain, and reliable assessment tools are available specifically for use with children. Opioid analgesics may be safely used with children as long as they are carefully monitored.

An RN in a long-term care facility supervises APs as they provide hygiene to older adults. What action by the AP will the nurse correct? A. When providing perineal care, washing the area from front to back B. Insisting the older adult must take a bath or shower each day C. Telling the patient to avoid soaking feet, helps the patient dry between the toes D. Covering areas not being bathed with a bath blanket

B. Insisting the older adult must take a bath or shower each day b. Older adults tend to develop dry skin; bathing frequency will change accordingly. Soaking adults' feet is not recommended. It is appropriate to keep a patient warm with bath blankets and provide perineal care washing from front to back.

During a feedback session on the updates to the electronic health record, nurses complain that they must use standard terminology rather than enter "free text" data in their own words. How will the nurses developing the system explain the purpose of standardization? A. It leads to fewer spelling errors and less misrepresentation of data. B. It provides for easier retrieval of data for use. C. The program was designed this way, and we cannot change it. D. Standardization uses preselected terminology, reducing lawsuits.

B. It provides for easier retrieval of data for use. b. Using standard terminology in the electronic health record allows data to be easily retrieved and better captures nursing's contribution to care delivery and patient outcomes. For example, to capture all patients using canes, quad canes, and walkers, a common phrase such as "uses assistive device" allow data for all these mobility issues to be captured.

A patient has a fractured left leg, which has been casted. Following teaching from the physical therapist for using crutches, the nurse reinforces which teaching point with the patient? A. Lean on the crutches using the axillae to bear body weight. B. Keep elbows close to the sides of the body. C. When rising, extend the uninjured leg to prevent weight bearing. D. To climb stairs, place weight on affected leg first.

B. Keep elbows close to the sides of the body. b. The patient should keep the elbows at the sides, prevent pressure on the axillae to avoid damage to nerves and circulation, extend the injured leg when rising to prevent weight bearing, and advance the unaffected leg first when climbing stairs.

A nurse is caring for a patient who has a pleural chest tube attached to a disposable chest drainage system. Which nursing actions are indicated for this patient? Select all that apply. A. Avoiding turning the patient to prevent disconnections in the tubing B. Maintaining an occlusive dressing on the site C. Assessing the patient for signs of respiratory distress D. Keeping the chest drainage device at the level of the patient's thorax E. Ensuring there are no dependent loops or kinks in the tubing F. Observing for bubbles indicating air leak in the water seal chamber

B. Maintaining an occlusive dressing on the site C. Assessing the patient for signs of respiratory distress E. Ensuring there are no dependent loops or kinks in the tubing F. Observing for bubbles indicating air leak in the water seal chamber b, c, e, f. The chest drainage collection device must be positioned below the tube's insertion site. Maintaining an occlusive dressing helps prevent air leak; assess for crepitus around the chest tube site indicating air leak. Avoid dependent loops or kinks in the tubing, which could impede drainage. Assess for bubbling in the water seal, maintaining the water level at the 2-cm mark. When a chest tube becomes separated from the drainage device, the nurse should submerge the tube's end in water, creating a temporary water seal and allowing air to escape until a new drainage unit can be attached.

A patient is receiving a multimodal medication regimen as part of the treatment plan for neuropathic phantom limb pain. When the patient reports a bloody bowel movement, which medication prescription requires notification of the provider? A. Acetaminophen B. Nonsteroidal antiinflammatory C. Opioid medication D. Antianxiety medication

B. Nonsteroidal antiinflammatory b. The NSAIDs are contraindicated in patients with bleeding disorders (their action may interfere with platelet function). The nurse will hold the medication and collaborate with the provider to adjust the patient's prescriptions.

A nurse in the emergency department is caring for a patient who had eaten shellfish and is now wheezing. The nurse explains to the patient that the health care provider has prescribed a bronchodilator, which will have what action? A. Helping the patient cough up thick mucus B. Opening narrowed airways and relieving wheezing C. Acting as a cough suppressant D. Blocking the effects of histamine

B. Opening narrowed airways and relieving wheezing b. A bronchodilator opens narrowed airways which result in wheezing. An expectorant encourage cough to clear secretions. A cough suppressant reduces, treats, or stops a cough. Medications that block histamine (antihistamine) are often used for allergy but are not specific bronchodilators.

A charge nurse in a skilled nursing facility is working to reduce patients' foot and nail problems. The charge nurse reminds the nurses and APs to closely observe which of these patients at higher risk? Select all that apply. A. Patient taking antibiotics for chronic bronchitis B. Patient with type 2 diabetes C. Patient who has obesity D. Patient who frequently bites their nails E. Patient with prostate cancer F. Patient who frequently washes their hands

B. Patient with type 2 diabetes C. Patient who has obesity D. Patient who frequently bites their nails F. Patient who frequently washes their hands b, c, d, f. Variables known to cause nail and foot problems include deficient self-care abilities, vascular disease, arthritis, diabetes mellitus, history of biting nails or trimming them improperly, frequent or prolonged exposure to chemicals or water, trauma, ill-fitting shoes, and obesity. Antibiotic use and prostate cancer do not predispose to foot or nail problems.

A nurse in the emergency department is caring for a patient who was brought in by fire rescue due to a heroin overdose. The nurse notes the patient is not breathing. What action will the nurse take immediately? Select all that apply. Tilt the patient's head forward. A. Begin ventilation using a manual resuscitation bag (Ambu bag). B. Place the mask tightly over the patient's nose and mouth. C. Pull the patient's jaw backward. D. Compress the bag twice the normal respiratory rate for the patient. E. Recommend that a sputum culture for cytology is obtained.

B. Place the mask tightly over the patient's nose and mouth. C. Pull the patient's jaw backward. b, c. The priority is to establish ventilation using the manual resuscitation bag to provide emergency or rescue breathing. The nurse tilts the head back, pulls the jaw forward, and positions the mask tightly over the patient's nose and mouth. The bag is compressed at a rate that approximates normal respiratory rate (e.g., 12 to 20 breaths/min in adults). Sputum for cytology is done primarily to detect cells that may be malignant, determine organisms causing infection, and identify blood or pus in the sputum. Note that the bag, with the mask removed, also fits easily over tracheostomy and endotracheal tubes.

A nurse is using informatics technology to determine which patients may be at risk for readmission. What term best describes this type of analytic? A. Data visualization B. Predictive analytics C. Big data D. Data recall

B. Predictive analytics b. Predictive analytics uses statistical techniques to analyze current and historical facts to make predictions about future or otherwise unknown events. This is used by health care organizations to identify patients who are at risk for readmission, allowing case managers to intervene. Data visualization presents data in a pictorial or graphical format for analysis. Big data comprises the accumulation of health care-related data from various sources, transforming the data to information, to knowledge, and ultimately to wisdom. Data recall is not a technical term for analytics.

A nurse is caring for a 25-year-old patient who is unresponsive following a head injury. The patient has several piercings in the ears and nose that appear crusted and slightly inflamed. What is the most appropriate action to care for this patient's piercings? A. Avoiding removing or washing the piercings until the patient is responsive B. Rinsing the sites with warm water and remove crusts with a cotton swab C. Washing the sites with alcohol and apply an antibiotic ointment D. Removing the jewelry and allow the sites to heal over

B. Rinsing the sites with warm water and remove crusts with a cotton swab b. When providing care for piercings, the nurse performs hand hygiene, applies gloves, then cleanses the site of all crusts and debris by rinsing the site with warm water and removing the crusts with a cotton swab. The nurse should then apply a dab of liquid-medicated cleanser, per policy, to the area, turn the jewelry back and forth to work the cleanser around the opening, rinse well, remove gloves, and perform hand hygiene. The nurse should not use alcohol, peroxide, or ointments at the site and should avoid removing piercings unless it is absolutely necessary (e.g., when an MRI is ordered.)

A nurse on a surgical unit has assessed and documented a patient's wound and drainage. Which statements most accurately describe the characteristic of the wound drainage? Graphic Record T 99.9 P100 RR 20 BP 138/88 Nursing note: Patient postoperative day 2. Dry sterile dressing changed on abdominal incision. Incision edges are well approximated with a slight ½-cm opening at inferior edge; incisional edges reddened. Hemovac draining sanguineous material, 60 mL for the shift. Patient reports moderate pain, relieved by oxycodone X1. ------------------------------- A. Sanguineous drainage is composed of the clear portion of the blood and serous membranes. B. Sanguineous drainage is composed of a large number of red blood cells and looks like blood. C. Sanguineous drainage is composed of white blood cells, dead tissue, and bacteria. D. Sanguineous drainage is thin, cloudy, and watery and may have a musty or foul odor.

B. Sanguineous drainage is composed of a large number of red blood cells and looks like blood. b. Sanguineous drainage consists of large numbers of red blood cells and looks like blood. Bright-red sanguineous drainage is indicative of fresh bleeding, whereas darker drainage indicates older bleeding. Serous drainage, generally watery, is composed primarily of the clear, serous portion of the blood and serous membranes. Purulent drainage is made up of white blood cells, liquefied dead tissue debris, and both dead and live bacteria. It is thick, often has a musty or foul odor, and varies in color (such as dark yellow or green), depending on the causative organism.

A nurse in a long-term care facility observes the AP providing foot care for patients. Which actions by the AP require the nurse to intervene? Select all that apply. A. Bathing the feet thoroughly in a mild soap and tepid water solution B. Soaking the resident's feet in warm water and bath oil C. Drying the feet and area between the toes thoroughly D. Applying an alcohol rub for odor and dryness to the feet E. Applying an antifungal foot powder F. Cutting the toenails at the lateral corners when trimming the nail

B. Soaking the resident's feet in warm water and bath oil D. Applying an alcohol rub for odor and dryness to the feet F. Cutting the toenails at the lateral corners when trimming the nail b, d, f. The nurse corrects the AP for soaking the feet or using alcohol and reminds them to use moisturizer if the feet are dry. Digging into or cutting the toenails at the lateral corners when trimming the nails requires correction; toenails should be trimmed straight across. Guidelines for foot care include bathing the feet thoroughly in a mild soap and tepid water solution; drying feet thoroughly, including the area between the toes; and applying an antifungal foot powder when requested.

After an initial skin assessment, the nurse documents the presence pressure area that is reddened and has a 1-cm blister. How will the nurse document the wound stage? A. Stage 1 dark maroon wound, skin intact B. Stage 2 with 1-cm blister noted C. Stage 3 wound base with red granulation tissue D. Stage 4 blanchable reddened area, 2 cm

B. Stage 2 with 1-cm blister noted b. A stage 2 pressure injury involves partial-thickness loss of dermis and presents as a shallow open ulcer with a red-pink wound bed, without slough. It may also present as an intact or open/ruptured serum-filled blister. Dark maroon or purple wounds with intact skin represent deep tissue injury. Red granulation tissue is present in stage 3 or 4 pressure injuries that are healing. A blanchable, red area is a stage 1 pressure injury.

A nurse working in the pulmonary clinic is providing teaching to patients with altered oxygenation due to conditions such as asthma and COPD. Which measures would the nurse recommend? Select all that apply. A. Avoid exercise. B. Take steps to manage or reduce anxiety. C. Eat meals 1 to 2 hours prior to breathing treatments. D. Eat a high-protein/high-calorie diet. E. Maintain a high-Fowler position when possible. F. Drink 2 to 3 pints of clear fluids daily.

B. Take steps to manage or reduce anxiety. D. Eat a high-protein/high-calorie diet. E. Maintain a high-Fowler position when possible. b, d, e. When caring for patients with COPD, it is important to help create an environment that is likely to reduce anxiety, which increases oxygen demand. A high-protein/high-calorie diet is recommended to meet increased energy needs due to the work of breathing. People with dyspnea and orthopnea are most comfortable in a high-Fowler (upright) position because accessory muscles can easily be used to facilitate respiration and lung expansion. Meals should be eaten 1 to 2 hours after breathing treatments; exercises and drinking 2 to 3 quarts (1.9 to 2.9 L) of clear fluids daily is recommended, rather than 2 to 3 pints.

A physician writes an order using the abbreviation MS. The order states "MS 10 mg IV push every 6 hours as needed for pain." According to The Joint Commission's "Do Not Use" list, what is the potential problem in this order? A. The order does not include a dosage. B. The drug could be magnesium sulfate or morphine sulfate. C. The potential problem is minimal, and the order is clear. D. The order does not include the route.

B. The drug could be magnesium sulfate or morphine sulfate.

A nurse is developing a care plan for an older adult patient who is recovering from a hip arthroplasty (hip replacement). Which assessment findings indicate a high risk for this patient to develop area(s) of pressure injury? Select all that apply. A. The patient takes time to think about responses to questions. B. The patient is an older adult with a poor appetite. C. The patient reports inability to control their urine. D. The patient's albumin level is <3.2 mg/dL (normal, 3.4 to 5.4 g/dL). E. Lab findings include BUN 12 (older adult, normal 8 to 23 mg/dL) and creatinine 0.9 (adult female, normal 0.61 to 1 mg/dL). F. The patient reports increased pain in right hip when repositioning in bed or chair.

B. The patient is an older adult with a poor appetite. C. The patient reports inability to control their urine. D. The patient's albumin level is <3.2 mg/dL (normal, 3.4 to 5.4 g/dL). F. The patient reports increased pain in right hip when repositioning in bed or chair. b, c, d, f. Pressure, friction, and shear, as well as other factors, usually combine to contribute to pressure injury development. The skin of older adults is more susceptible to injury; incontinence contributes to prolonged moisture on the skin, as well as negative effects related to urine in contact with skin; hip surgery involves decreased mobility during the postoperative period, as well as pain with movement, contributing to immobility; and increased pain in the hip may contribute to increased immobility. A low albumin level signals a risk for poor wound healing related to malnutrition. Apathy, confusion, and/or altered mental status are risk factors for pressure injury development; however, taking time to formulate responses is consistent with normal aging. This patient's BUN and creatinine are within normal range; however, dehydration (indicated by an elevated BUN and creatinine) is a risk for pressure injury development.

A nurse is scheduling hygiene for patients on the unit. What is the priority the nurse uses to guide planning for patient's personal hygiene? A. When the patient had their most recent bath B. The patient's usual hygiene practices and preferences C. Where the bathing fits in the nurse's schedule D. The time that is convenient for the AP

B. The patient's usual hygiene practices and preferences b. The patient's preferences, practices, and rituals should always be taken into consideration, unless there is a clear threat to health. The patient and nurse should work together to come to a mutually agreeable time and method to accomplish the patient's personal hygiene. The availability of staff to assist may be important, but the patient's preferences are a higher priority.

A nursing student asks the primary nurse why an immobile patient developed two urinary tract infections (UTIs) in the 6 months. How does the nurse best explain this patient's risk for UTI? A. Improved renal blood supply to the kidneys B. Urinary stasis C. Decreased urinary calcium D. Acidic urine formation

B. Urinary stasis b. In a nonerect patient, the kidneys and ureters are level, limiting or delaying urinary drainage from the kidney pelvis to ureter and bladder. The resulting urinary stasis favors the growth of bacteria that can promote urinary tract infections. Regular exercise, not immobility, improves blood flow to the kidneys. Immobility predisposes the patient to bone demineralization, resulting in increased urinary calcium levels and alkaline urine, contributing to renal calculi and urinary tract infection, respectively.

A nurse assisting a patient with a bed bath observes the older adult has dry skin, which the patient states is "itchy." Which intervention is appropriate? A. Bathe the patient more frequently. B. Use an emollient on the dry skin. C. Explain that this is expected as people age. D. Limit the patient's fluid intake.

B. Use an emollient on the dry skin. b. An emollient soothes dry skin, whereas frequent bathing increases dryness. Telling the patient this is normal with aging and does not help resolve the issue. Limiting fluid intake can promote dehydration and exacerbate dry skin.

A patient reports diffuse abdominal pain that is difficult to localize. The nurse documents this as which type of pain? A. Cutaneous B. Visceral C. Superficial D. Somatic

B. Visceral b. Visceral pain, which is poorly localized and can originate in body organs in the abdomen. Cutaneous pain (superficial pain) usually involves the skin or subcutaneous tissue. A paper cut that produces sharp pain with a burning sensation is an example of cutaneous pain. Deep somatic pain is diffuse or scattered and originates in tendons, ligaments, bones, blood vessels, and nerves. Strong pressure on a bone or damage to tissue that occurs with a sprain causes deep somatic pain.

A patient with an overdose of an oral drug usually receives which of the following? A. specific antidote B. activated charcoal C. syrup of ipecac D. strong laxative

B. activated charcoal Activated charcoal is considered the "universal antidote." Most therapeutic drugs do not have specific antidotes; syrup of ipecac to induce vomiting with overdoses of some drugs is no longer recommended (questionable effectiveness, danger of aspiration of vomitus), and strong laxatives are not indicated.

Students in a leadership class are discussing how social determinants of health affect pain management. Which statement is correct and should be included in the discussion? A. "Outcomes of pain management are generally satisfactory regardless of income." B. "Minority patients often receive excess medication." C. "Patients from minority groups often wait a long time before seeking treatment for pain." D. "Social determinants of health are unrelated to pain management."

C. "Patients from minority groups often wait a long time before seeking treatment for pain." c. Social determinants of health greatly impact on pain management outcomes. Patients residing in low-income settings and minority populations are less likely to receive the recommended treatments for pain in health care settings. Minority patients often experience pain for a lengthy period of time before seeking treatment for it. Language barriers, culturally inappropriate pain assessment tools, and prejudice and misconceptions may contribute to unsatisfactory pain outcomes. Initiatives to address health care access and treatment disparities begin at the local and community level.

A nurse is explaining to a patient the anticipated effect of the application of cold to an injured area. What response indicates that the patient understands the explanation? A. "There will be more discomfort in the area where the cold is applied." B. "I should expect more drainage from the incision after the ice has been in place." C. "Redness and swelling should decrease after cold treatment." D. "My incision may bleed more when the ice is first applied."

C. "Redness and swelling should decrease after cold treatment." c. The local application of cold constricts peripheral blood vessels, reduces muscle spasms, and promotes comfort. Cold reduces blood flow to tissues, decreases the local release of pain-producing substances, decreases metabolic needs, and capillary permeability. The resulting effects include decreased edema, coagulation of blood at the wound site, promotion of comfort, decreased drainage from wound, and decreased bleeding.

A nurse is providing education to a patient and their family regarding the use of negative pressure wound therapy (NPWT). The nurse documents that the teaching has been effective when the patient and family make which statement? A. "This therapy is used to collect excess blood loss and prevent formation of a scab." B. "The suction created will prevent infection and promote wound healing with less scar tissue." C. "Suction stimulates blood flow to the wound, removes excess fluid, and promotes a moist environment for healing." D. "This treatment irrigates the wound, suctions the irrigation fluid from the wound, and keeps it free from debris wound exudate."

C. "Suction stimulates blood flow to the wound, removes excess fluid, and promotes a moist environment for healing." c. Negative pressure wound therapy (NPWT) promotes wound healing and wound closure through the application of uniform negative pressure on the wound bed, reduction in bacteria in the wound, and the removal of excess wound fluid, while providing a moist wound healing environment. The negative pressure results in mechanical tension on the wound tissues, stimulating cell proliferation, blood flow to wounds, and the growth of new blood vessels. It is used to treat a variety of acute or chronic wounds, wounds with heavy drainage, wounds failing to heal, or healing slowly.

A nurse is getting a patient with right hemiparesis out of bed to the chair. What will the nurse say to the patient? A. "Stand on the weaker leg and pivot toward the chair." B. "I will call the lift team to carry you to the chair." C. "The chair is by your non-affected leg for smoother movement." D. "Avoid putting your hospital socks on, as that will restrict your feet moving."

C. "The chair is by your non-affected leg for smoother movement." c. When transferring a patient, the chair is placed on the unaffected or stronger side, rather than the weaker or affected side. Lifting and carrying a patient unless absolutely necessary poses an unnecessary risk for injury to patient and staff. Patients should wear proper shoes, sturdy slippers, or hospital-issued socks with grips to prevent sliding and/or falling.

A patient is to receive lamotrigine 300 mg by mouth two times per day. The pharmacy has delivered 50-mg tablets. How many tablets should the nurse administer each time? A. 2 tablets B. 4 tablets C. 6 tablets D. 8 tablets

C. 6 tablets . C. The nurse administers six tablets, for a total of 300 mg.

A nurse plans to suction a patient's endotracheal tube using the open suction technique. Which intervention is appropriate for this technique? A. Using a suction catheter that is the diameter of the endotracheal tube B. Maintaining the patient in the supine position C. Administering oxygen prior to suctioning D. Changing the inline suction device every 24 hours

C. Administering oxygen prior to suctioning c. To prevent hypoxemia, prior to endotracheal suctioning, the nurse provides 100% oxygen for a minimum of 30 seconds. This is referred to as hyperoxygenation. The nurse limits the application of suction to no more than 10 to 15 seconds (AARC, 2010; Burns & Delgado, 2019; Hess et al., 2021; Pasrija & Hall, 2020). The external diameter of the suction catheter should not exceed half of the internal diameter of the endotracheal tube. An inline suction device is considered a closed, self-contained system used for a "closed technique" for suction; these are changed every 24 hours.

A community health nurse is providing education on child safety. Who does the nurse identify as at highest risk for choking and suffocation? A. A toddler playing with his older brother's wooden blocks B. A 4-year-old eating yogurt and strawberries for lunch C. An infant sleeping in the prone position D. A 3-year-old drinking a glass of juice

C. An infant sleeping in the prone position c. Infants should be placed on their backs to sleep. A young child may place small or loose parts in the mouth. Anything that will fit through the average toilet paper roll is not safe for a toddler. A 3-year-old and a 4-year-old drinking juice and eating yogurt are developmentally appropriate.

The nurse preceptor is supervising a new graduate nurse as they assess a patient with a pressure injury. The graduate nurse documents the presence of biofilm in the wound. The preceptor recognizes the graduate nurse understands this concept when the graduate makes which of these statements? Select all that apply. A. Enhanced healing occurs due to the presence of sugars and proteins. B. Delayed healing develops due to dead tissue present in the wound. C. Antibiotics against the bacteria become less effective. D. Skin loses its integrity due to overhydration of the cells of the wound. E. Delayed healing due to cells dehydrating and dying occurs. F. Decreased effectiveness of the patient's normal immune process results.

C. Antibiotics against the bacteria become less effective. F. Decreased effectiveness of the patient's normal immune process results. c, f. Wound biofilms are the result of wound bacteria growing in clumps, embedded in a thick, self-made, protective, slimy barrier of sugars and proteins. This barrier contributes to decreased effectiveness of antibiotics against the bacteria (antibiotic resistance) and decreases the effectiveness of the normal immune response by the patient (Baranoski & Ayello, 2020). Necrosis (dead tissue) in the wound delays healing. Maceration or overhydration of cells related to urinary and fecal incontinence can lead to impaired skin integrity. Desiccation is the process of drying up, in which cells dehydrate and die in a dry environment.

When developing the care plan for a patient with chronic pain, the nurse plans interventions based on the knowledge that chronic pain not related to cancer or palliative/end-of-life care is most effectively relieved through which method? A. Providing the highest effective dose of an opioid on a PRN (as needed) basis B. Using nonopioid drugs conservatively C. Applying multimodal nonpharmacologic and nonopioid pharmacologic therapies D. Administering a continuous intravenous infusion on a regular basis

C. Applying multimodal nonpharmacologic and nonopioid pharmacologic therapies c. Nonpharmacologic and nonopioid pharmacologic therapies (multimodal) are the preferred choices for chronic pain that is unrelated to active cancer, palliative care, or end-of-life care. If progression to opioids becomes necessary, the lowest effective dose of an immediate-release opioid should be initiated first. Ongoing assessment and careful monitoring should guide the prescription of opioids for the management of chronic pain (Dowell et al., 2016). A PRN (as needed) drug regimen has not been proven effective for people experiencing chronic or acute pain. When caring for a patient with acute pain, such as postoperative pain, medication should be offered or requested before pain becomes severe or unbearable. Once pain is adequately treated, such as later in the postoperative course, a PRN schedule may be effective.

The nurse is administering the first dose of an anti-infective agent. Which of the following assessments should the nurse make prior to administering the anti-infective agent? A. Assess the patient's temperature. B. Assess the patient's level of consciousness. C. Assess whether the patient is allergic to any anti-infective agent. D. Assess whether the patient has taken the medication previously.

C. Assess whether the patient is allergic to any anti-infective agent. C. Prior to administering any medication, the nurse assesses the client's allergies. Clients who are allergic to penicillins are also likely to be allergic to cephalosporins.

A home care nurse is assisting an older adult with an unsteady gait with a tub bath. Which action is recommended in this procedure? A. Adding bath oil to the water to prevent dry skin B. Allowing the patient to lock the door to guarantee privacy C. Assisting the patient in and out of the tub to prevent falling D. Keeping the water temperature very warm because older adults chill easily

C. Assisting the patient in and out of the tub to prevent falling c. Safe nursing practice requires that the nurse assists a patient with an unsteady gait in and out of the tub. Adding bath oil to the bath water poses a safety risk because it makes the patient and tub slippery. Although privacy is important, if the patient locks the door, the nurse cannot help if there is an emergency. The water should be comfortably warm at 43° to 46°C. Older adults have an increased susceptibility to burns due to diminished sensitivity.

A patient was in an automobile accident and received a wound across the nose and cheek. After surgery to repair the wound, the patient says, "I am so ugly now." Based on this statement, psychosocial problem will the nurse plan to address? A. Pain B. Wound healing C. Body image D. Change in cognition

C. Body image c. Wounds cause emotional as well as physical stress.

A school nurse is teaching about adolescent safety with students entering high school. What will the nurse include in the discussion about the major causes of death in this group? Select all that apply. A. Choking B. Diving accidents C. Car accidents D. Suicide E. Intimate partner violence F. Cigarette smoking

C. Car accidents D. Suicide c, d. Car accidents and suicide are common causes of death in adolescents. Choking is more typical in children younger than age 3 years. While diving accidents can occur in adolescents due to poor judgment, this is not as common. Intimate partner violence is more common in adults. Smoking, while ill advised, takes many years or decades to become a cause of death.

A nurse designing a new EHR system for a pediatric office follows usability concepts in system design. Which concepts are recommended in system design? Select all that apply. A. Users should not explore with forgiveness for unintended consequences. B. Shortcuts for frequent users should not be incorporated into the system. C. Content emphasis should be on information needed for decision making. D. The less times users need to apply prior experience to a new system the better. E. All the information needed should be presented to reduce cognitive load. F. The number of steps it takes to complete tasks should be minimized.

C. Content emphasis should be on information needed for decision making. E. All the information needed should be presented to reduce cognitive load. F. The number of steps it takes to complete tasks should be minimized. c, e, f. When designing a system, content emphasis should be on information needed for decision making. All the information needed should be presented to reduce cognitive load. The number of steps it takes to complete tasks should be minimized. The more users can apply prior experience to a new system, the lower the learning curve, the more effective their usage, and the fewer their errors. Forgiveness means that a design allows the user to discover it through exploration without fear of disastrous results or unintended consequences. Minimizing the number of steps needed to complete tasks and to provide shortcuts for use by frequent and/or experienced users facilitates efficient user interactions.

A nurse plans to promote a patient's natural pain mediators by using a whirlpool following intensive physical therapy to the legs. What is a potent pain-blocking neuromodulator, released through relaxation techniques? A. Prostaglandins B. Substance P C. Endorphins D. Serotonin

C. Endorphins c. Endorphins are produced at neural synapses at various points along the CNS pathway. They are powerful pain-blocking chemicals that have prolonged analgesic effects and produce euphoria. It is thought that endorphins are released through pain relief measures, such as relaxation techniques. Prostaglandins, substance P, and serotonin (a hormone that can act to stimulate smooth muscles, inhibit gastric secretion, and produce vasoconstriction) are neurotransmitters or substances that either excite or inhibit target nerve cells.

A nurse is assisting a patient who is 2 days postoperative from a cesarean section dangle in preparation for sitting in a chair. After assisting the patient to stand up, the patient's knees buckle and she tells the nurse she feels faint. What is the appropriate nursing action? A. Supporting the patient as she stands, waiting a few moments, then continuing the move to the chair B. Calling for assistance and continuing the move with the assistance of another nurse C. Lowering the patient back to the side of the bed and pivoting her back into bed D. Having the patient sit down on the bed and dangle her feet before moving

C. Lowering the patient back to the side of the bed and pivoting her back into bed c. If a patient becomes faint and their knees buckle when moving from bed to a chair or ambulating, the nurse should stop the activity, as the patient has demonstrated a clear risk for falling. The nurse should lower the patient back to the side of the bed, pivot her back into bed, cover her, and raise the side rails. Assess the patient's vital signs and for the presence of other symptoms. When vital signs are stable, another attempt can be made with the assistance of another staff. Instruct the patient to remain in the sitting position on the side of the bed for several minutes to allow the circulatory system to adjust to a change in position and prevent hypotension related to a sudden change from the supine position.

A nurse wants to be an ethical practitioner. When using informatics, which action best represents ethical practice? A. Selecting appropriate standardized care plans to meet the patient's needs B. Gathering data for research studies using the electronic health record of multiple patients C. Maintaining confidentiality of patient records within legal and regulatory parameters D. Carrying out all electronic orders entered by the health care provider

C. Maintaining confidentiality of patient records within legal and regulatory parameters c. The ANA asserts ethical use of informatics principles, standards, and methodologies to establish and maintain health care consumer confidentiality be used within legal and regulatory parameters.

A school nurse is teaching parents about home and fire safety. What information will be included in the teaching plan? Select all that apply. A. Sixty percent of U.S. fire deaths occur in the home. B. Most fatal fires occur when people are cooking. C. Most people who die in fires die of smoke inhalation. D. Fire-related injury and death have declined due to the availability and use of smoke alarms. E. Fires are more likely to occur in homes without electricity or gas. F. Fires are less likely to spread if bedroom doors are kept open when sleeping.

C. Most people who die in fires die of smoke inhalation. D. Fire-related injury and death have declined due to the availability and use of smoke alarms. E. Fires are more likely to occur in homes without electricity or gas. c, d, e. Eighty percent of fire deaths in the United States occur in the home. Most fatal home fires occur while people are sleeping, and most deaths result from smoke inhalation rather than burns. The widespread availability and use of home smoke alarms is considered the primary reason for the decline in fire-related injury and death. People with limited financial resources may use space or kerosene heaters, wood stoves, or a fireplace as the sole source of heat if utilities are turned off. Bedroom doors should be kept closed when sleeping and monitors used to listen for children.

While discussing home safety with the nurse, a patient admits that they smoke a cigarette in bed before falling asleep at night. Which health problem is the priority for this patient? A. Impaired gas exchange: etiology, cigarette smoking B. Acute anxiety: etiology, inability to stop smoking C. Nonadherence: etiology, nonadherence to recommendation to stop smoking D. Knowledge deficiency: etiology, risk for burn and suffocation in a house fire

C. Nonadherence: etiology, nonadherence to recommendation to stop smoking c. Because the patient is not aware or denies that smoking in bed poses a danger for fire and toxic fumes, education about the risk for burns and suffocation is needed. The other three nursing diagnoses are correctly stated but are not a priority in this situation.

A nurse assists a patient with ambulation for the first time following a knee replacement. Shortly after beginning to walk, the patient tells the nurse that they are dizzy and feel like they might fall. Place these nursing actions in the order in which the nurse should perform them to protect the patient: A. Grasp the gait belt. B. Stay with the patient and call for help. C. Place feet wide apart with one foot in front. D. Gently slide the patient down to the floor, protecting their head. E. Pull the weight of the patient backward against your body. F. Rock your pelvis out on the side of the patient.

C. Place feet wide apart with one foot in front. F. Rock your pelvis out on the side of the patient. A. Grasp the gait belt. E. Pull the weight of the patient backward against your body. D. Gently slide the patient down to the floor, protecting their head. B. Stay with the patient and call for help. c, f, a, e, d, b. When a patient is being moved or ambulated and starts to fall, the nurse places their feet wide apart with one foot in front, rocks their pelvis out toward the side of the patient, grasps the gait belt, supports the patient by pulling the patient's weight backward against their body, gently slides the patient down their body toward the floor while protecting the patient's head, and remains with the patient while calling for help.

A nurse on a medical-surgical unit notes a patient with pneumonia and is experiencing dyspnea. What action will the nurse take to improve the dyspnea? A. Encourage the patient to ambulate. B. Suggest the patient use music or television as distraction. C. Place the patient in Fowler's position. D. Tell the patient to take several deep breaths, then hold their breath for 5 seconds.

C. Place the patient in Fowler's position. c. High-Fowler's position promotes maximal lung expansion and is the position of choice during episodes of dyspnea. Encouraging ambulation during distress will increase dyspnea. Distracting the patient is not addressing the underlying cause of dyspnea, which is activity. Holding the breath increases demands on the heart.

A postoperative patient who has a large abdominal incision suddenly calls out for help, shouting, "Something is falling out of my incision!" The nurse notes the wound is gaping open with tissue bulging outward. Place the nursing interventions in the order they should be performed, arranged from first to last. A. Notify the health care provider of the situation. B. Cover exposed tissue with sterile towels moistened with sterile 0.9% sodium chloride solution. C. Place the patient in the low Fowler position. D. Document the findings and outcome of interventions. E. Maintain NPO status for return to the OR for repair.

C. Place the patient in the low Fowler position. B. Cover exposed tissue with sterile towels moistened with sterile 0.9% sodium chloride solution. A. Notify the health care provider of the situation. E. Maintain NPO status for return to the OR for repair. D. Document the findings and outcome of interventions. c, b, a. e. d. The correct order of nursing interventions for this postoperative emergency is to place the patient in the low Fowler position (to prevent further damage or protrusion from increased intraabdominal pressure), cover exposed tissue with sterile towels moistened with sterile 0.9% sodium chloride solution (to protect the viscera), and notify the health care provider of the situation (to address the issue, likely with surgery). The patient is kept NPO, as prompt surgical repair will be needed. After the patient has received attention, the nurse documents all assessments and interventions in a timely manner.

A nurse is monitoring a patient with a pleural effusion after a thoracentesis removing 1,400 mL of dark yellow liquid. What is the expected outcome of this procedure? A. Tachycardia B. Hypotension C. Reduced dyspnea D. Pulse oximetry of 88%

C. Reduced dyspnea c. Thoracentesis involves inserting a needle into the pleural space to aspirate pleural fluid, air, or both (Morton & Fontaine, 2018). A thoracentesis may be performed to obtain a specimen for diagnostic purposes, to remove fluid or air that has accumulated in the pleural cavity and is causing respiratory difficulty and discomfort, or to instill medications (Hinkle et al., 2022).

A nurse is securing a patient's endotracheal tube with tape and observes that the tube depth changed during the retaping. Which action would be appropriate related to this incident? A. Instructing the assistant to notify the health care team B. Assessing the patient's vital signs C. Removing the tape, adjusting the depth to the ordered depth, and retaping securely D. Taking no action, as the depth will adjust automatically

C. Removing the tape, adjusting the depth to the ordered depth, and retaping securely c. The tube depth should be maintained at the same level unless otherwise prescribed. If the depth changes, the nurse should remove the tape or securement device, adjust the tube to the ordered depth, and reapply the tape or securement device.

A nurse is about to bathe a female patient who has an IV in the forearm. The patient's gown, which does not have snaps on the sleeves, needs to be removed prior to bathing. How will the nurse proceed? A. Quickly disconnecting the IV tubing closest to the patient and thread it through the gown sleeve B. Cutting the gown with scissors to allow arm movement C. Threading the bag and tubing through the gown sleeve, keeping the line intact D. Temporarily disconnecting the tubing from the IV container, threading it through the gown

C. Threading the bag and tubing through the gown sleeve, keeping the line intact c. Threading the bag and tubing through the gown sleeve maintains a closed system and prevents contamination. No matter how quickly performed, any disconnection of IV tubing results in a breach of the sterile system, creating risk for infection. Cutting a gown is not an alternative except in an emergency.

Nurses test new technology in phases. When will the nurses plan to "test drive" the new system? A. Unit B. Function C. User acceptance D. Integration

C. User acceptance c. During the phase "user acceptance," the nurse would "test drive" the new system to ensure it's working as designed. Unit testing is basic testing that occurs initially. Function testing uses test scripts to validate that a system is working as designed for one particular function. Integration testing uses test script to validate that a system is working as designed for an entire workflow that integrates multiple components of the system.

A nurse in a memory care unit is assisting a patient with dementia with bathing. Which nursing action will enhance patient comfort and prevent anxiety? A. Shifting the focus of the interaction to the "process of bathing" B. Washing the face and hair at the beginning of the bath C. Using music to soothe anxiety and agitation D. Avoiding towel baths or forms of bathing with which the patient is unfamiliar

C. Using music to soothe anxiety and agitation c. The nurse use music to soothe anxiety and agitation. The nurse should also shift the focus of the interaction from the "task of bathing" to the needs and abilities of the patient, and focus on comfort, safety, autonomy, and self-esteem, in addition to cleanliness. Wash the face and hair at the end of the bath or at a separate time. Water dripping in the face and having a wet head are often the most upsetting parts of the bathing process for people with dementia. The nurse should also consider methods for bathing aside from showers and tub baths. Towel baths, washing under clothes, and bathing "body sections" one day at a time, as well as dry shampoo or "shower cap" shampoos, are additional options.

A prescriber has written an order for an oral antihypertensive medication for a patient who is in rehabilitation following a stroke. Prior to administering the medication, which of the following nursing interventions is most important? (Select all that apply.) A. allowing the patient to take the medication with thickened liquids B. placing the patient in the sitting position C. assessing the patient's blood pressure D. assessing the patient's ability to swallow

C. assessing the patient's blood pressure D. assessing the patient's ability to swallow The nurse should assess the patient's blood pressure prior to administering the antihypertensive agent. If the blood pressure is less than 90 systolic or less than 60 diastolic, the medication should be held. The nurse should report the low blood pressure to the prescriber. A client who has had a stroke can suffer from dysphagia, the inability to swallow. The nurse must assess the client's ability to swallow prior to administering an oral medication. If the client has dysphagia and the medication is administered, the client is at risk for aspiration and pneumonia.

How do nursing interventions increase safety and effectiveness of drug therapy? A. by avoiding the use of nondrug measures during drug therapy B. by using multiple drugs to relieve most symptoms or problems C. by teaching patients about their drug therapy D. by avoiding excessive instructions

C. by teaching patients about their drug therapy C. Teaching clients about drug therapy increases safety in drug therapy.

A nurse is reading a research report about use of a medication that describes the pharmacokinetics of a particular medication that a patient is taking. Pharmacokinetics involves A. drug effects on human cells B. drug binding with receptors C. drug absorption, distribution, metabolism, and elimination D. drug stimulation of normal cell functions

C. drug absorption, distribution, metabolism, and elimination Pharmacokinetics involves movement of a drug through the body and its elimination. Drug binding with receptors and drug effects on cells are pharmacodynamic processes. Drug stimulation of normal cell functions describes an agonist drug.

A nurse is caring for a man who has worsening liver disease. In monitoring his medication, it is important to know that a patient with liver disease may have impaired drug A. absorption B. distribution C. metabolism D. excretion

C. metabolism The liver is mainly involved in drug metabolism. The gastrointestinal tract is mainly involved in absorption of oral drugs, the cardiovascular system is concerned with distribution, and the kidneys are concerned with excretion.

A nurse is administering an elixir. Which of the following measures is appropriate? A. microgram B. milligram C. milliliter D. kilogram

C. milliliter C. An elixir is a liquid medication that is measured in milliliters (mL).

A patient is asking what the difference is between a prescription for 800 mg of a medication that can be purchased on an OTC basis as a 200-mg tablet. To address this issue, it is important that the nurse knows that OTC drugs A. are considered safe for any consumer to use B. are not available for treatment of most commonly occurring symptoms C. often differ in indications for use and recommended dosages from their prescription versions D. are paid for by most insurance policies

C. often differ in indications for use and recommended dosages from their prescription versions Uses may be different, and recommended dosages of over-the-counter (OTC) drugs are usually lower than prescription versions. Labels of OTC drugs contain a listing of people who should not use the drugs, and the drugs are not safe for everyone. Many OTC drugs are available to treat cold symptoms, heartburn, constipation, and other common problems. Insurance companies do not pay for OTC drugs.

The nurse is caring for a woman who has strong beliefs about not putting anything unnatural into her body. It is most accurate to say that most modern medications are A. natural products derived from plants B. natural products derived from minerals C. synthetic products manufactured in laboratories D. synthetic modifications of natural products

C. synthetic products manufactured in laboratories Although most older drugs originated as plants, minerals, or other natural products, newer drugs are mainly synthetic. Synthetic drugs are standardized and therefore more consistent in their characteristics and effects than natural products.

Which question by the nurse will be most helpful in determining whether a patient who is experiencing a myocardial infarction has referred pain? A. "Did your chest pain last 2 minutes or less? B. "Was the pain on the surface of your chest?" C. "Is this pain in your residual limb shooting or burning?" D. "Are you having any arm or shoulder pain?"

D. "Are you having any arm or shoulder pain?" d. Referred pain is perceived in an area distant from its point of origin, whereas transient pain is brief and passes quickly. One example is the pain of MI (heart attack) that can be felt in the shoulder or chest, among other areas. Superficial pain originates in the skin or subcutaneous tissue. Phantom pain may occur in a person who has had a body part amputated, either surgically or traumatically.

A nurse is teaching a patient how to use a metered-dose inhaler for asthma. Which comments from the patient assure the nurse that the teaching has been effective? Select all that apply. A. "I'll be careful not to shake the canister before using it." B. "It's important to hold the canister upside down when using it." C. "I have to remember to inhale the medication through my nose." D. "I will continue to inhale when the cold propellant is in my throat." E. "I won't inhale more than one spray with one breath." F. "I will activate the device while continuing to inhale."

D. "I will continue to inhale when the cold propellant is in my throat." E. "I won't inhale more than one spray with one breath." F. "I will activate the device while continuing to inhale." d, e, f. Common mistakes that patients make when using MDIs include failing to shake the canister, holding the inhaler upside down, inhaling through the nose rather than the mouth, inhaling too rapidly, stopping the inhalation when the cold propellant is felt in the throat, failing to hold their breath after inhalation, inhaling two sprays with one breath, and not activating the device while inhaling.

A postoperative patient asks the nurse about pain management following surgery. What teaching will the nurse provide? A. "Avoid asking for pain medication often, as it can be addictive." B. "It is better to wait until the pain is severe before asking for pain medication." C. "It's natural to have pain after surgery; it will lessen in intensity in a few days." D. "You will be more comfortable if you take the medication at regular intervals."

D. "You will be more comfortable if you take the medication at regular intervals." d. While many analgesics are ordered on a PRN (as needed) basis, patients should be taught that it is more difficult to relieve pain that prevent it. The patient should not wait until pain is severe or unbearable to request pain medication. Few people become addicted to the medications if used for a short period of time. Pain following surgery can be controlled and should not be considered a natural part of the experience that will lessen in time.

A nurse is preparing to administer a subcutaneous injection. What size needle should the nurse use to administer the injection? A. 18 gauge B. 20 gauge C. 23 gauge D. 25 gauge

D. 25 gauge D. The nurse administers a subcutaneous injection with a 25-gauge syringe.

The mother of a 14-month-old girl calls a nurse working in a pediatric clinic and reports that her daughter ingested an unknown number of sleeping pills about 4 hours ago and is now drowsy. The mother asks what she should do. The best response to give the mother is A. "Administer a dose of syrup of ipecac to ensure vomiting" B. "Call the Poison Control Center immediately" C. "Administer a strong laxative and observe for a response" D. "Call 911 to transport your daughter to the nearest emergency department"

D. Call 911 to transport your daughter to the nearest emergency department The main goals of treatment are starting treatment as soon as possible after drug ingestion, supporting and stabilizing vital functions, preventing further damage from the toxic agent by reducing absorption or increasing elimination, and administering antidotes when available and indicated. Given the time since ingestion and the drowsiness of the girl, this is best accomplished by emergency medical personnel.

A nurse working in a pediatrician's office receives calls from parents whose children have ingested a toxic substance from under the sink. How will the nurse advise the parents? A. Administer activated charcoal in tablet form and take child to the ED. B. Administer syrup of ipecac and take child to the ED. C. Bring the child in to the primary care provider for gastric lavage. D. Call the PCC immediately before attempting any home remedy.

D. Call the PCC immediately before attempting any home remedy. d. The nurse tells the parents to call the PCC immediately, before attempting a home remedy. Parents may be instructed to bring the child to an emergency facility for immediate treatment. Activated charcoal is not appropriate to use at home but under medical supervision, after the risks and benefits have been assessed. Syrup of ipecac is no longer recommended because vomiting may exacerbate the hazard as it vomited up. Gastric lavage is no longer prescribed routinely for the treatment of ingestion of a toxic substance because it may propel the poison into the small intestine, where absorption will occur. The amount of toxin removed by gastric lavage is relatively small.

A nurse is caring for a patient with an eye infection with a moderate amount of discharge. What is the most appropriate technique for the nurse to use when cleansing this patient's eyes? A. Using diluted hydrogen peroxide on a clean washcloth to wipe the eyes B. Wiping the eye from the outer canthus toward the inner canthus C. Positioning the patient on the opposite side of the eye to be cleansed D. Cleansing the eye using a different section of the cloth for each stroke until clean

D. Cleansing the eye using a different section of the cloth for each stroke until clean d. The nurse applies gloves for the cleaning procedure, uses water or normal saline, and a clean washcloth or gauze to cleanse the eyes. After dampening a cleaning cloth with the solution of choice, the nurse wipes once while moving from the inner canthus to the outer canthus of the eye to reduce forcing debris into the area drained by the nasolacrimal duct. The nurse should turn the cleansing cloth and use a different section for each stroke until the eye is clean.

During the admission process, a nurse orients an older adult to their hospital room. What is the current safety priority? A. Explaining how to use the telephone B. Introducing the patient to their roommate C. Reviewing the hospital policy on visiting hours D. Demonstrating how to operate the call bell

D. Demonstrating how to operate the call bell d. Teaching the patient to use the call bell is a safety priority; knowing how to use the phone, meeting the roommate, and knowledge of visiting hours will not necessarily prevent an accidental injury.

Nursing students are invited to participate in the clinical agency's annual disaster drill, simulating the release of an airborne infectious agent and ensuing panic. Which assignment is most appropriate for the students? A. Cleansing and dressing wounds sustained during the panic B. Triaging patients with respiratory symptoms and traumatic injuries C. Providing information to families of missing loved ones D. Ensuring everyone entering and working has an N95 mask

D. Ensuring everyone entering and working has an N95 mask d. Rapid assessment and triage are essential during a disaster. Delegating tasks appropriate to students are based on skill level and ability to complete skills independently. Ensuring masks are worn and the education for this, if needed, are within the educational and clinical skills of nursing students.

A nurse is maintaining airway patency in an unconscious patient by providing frequent nasopharyngeal suction. When would the nurse anticipate inserting a nasopharyngeal airway (nasal trumpet)? A. Vomiting during suctioning occurs. B. Secretions appear to contain stomach contents. C. The suction catheter touches an unsterile surface. D. Epistaxis is noted with continued suctioning.

D. Epistaxis is noted with continued suctioning. d. Repeated suctioning may injure or traumatize the nares, resulting in nosebleed (epistaxis). The nurse would recommend insertion of a nasal trumpet, which will facilitate suction while protecting the nasal mucosa from further trauma.

A disoriented older resident likes to wander the halls of their long-term care facility but becomes agitated when they cannot find their room. Which action is most appropriate as an alternative to restraints? A. Placing them in a geriatric chair near the nurses' station B.Using the sheets to secure them snugly in the bed C. Keeping the bed in the high position D. Identifying their door with his photograph and a balloon

D. Identifying their door with his photograph and a balloon d. This allows the resident to be on the move and be more likely to find their room when they want to return. Many facilities use this kind of approach, rather than restrict patients' movements. Identifying the patient's door with their photo and a balloon may resolve the issue without restraints. Using the geriatric chair and sheets are forms of physical restraint. Leaving the bed in the high position is a safety risk and would likely result in a fall.

A nurse is performing oral care on a patient who has advanced dementia. The nurse notes that the mouth is extremely dry with crusts remaining after the oral care. What action will the nurse take next? A. Recommend a consultation with an oral surgeon. B.Communicate the condition to the health care team. C. Gently scrape the oral cavity with a tongue depressor. D. Increase the frequency of the oral hygiene and apply mouth moisturizer to oral mucosa.

D. Increase the frequency of the oral hygiene and apply mouth moisturizer to oral mucosa. d. If initial oral care results in continued dryness of the oral cavity with crusting, the nurse should increase frequency of oral hygiene, apply mouth moisturizer to oral mucosa, and monitor fluid intake and output to ensure adequate intake of fluid. It is not necessary to report this condition prior to providing the interventions mentioned above; however, mouth care and re-evaluation of the oral cavity is documented. The crusts should not be scraped with a tongue depressor.

A nursing student who is planning their career trajectory tells the nurse preceptor how much they enjoy working with the electronic health record and informatics. What type of graduate program could the preceptor suggest? A. Clinical nurse specialist B. Nurse leader C. Superuser D. Informatics nurse specialist (INS)

D. Informatics nurse specialist (INS) d. The INS is a registered nurse with formal graduate-level education in informatics. This nurse participates in strategy development, implementation, and maintenance and evaluation of clinical systems, in collaboration with multiple disciplines. The information nurse (IN) is a registered nurse with an interest or experience in informatics, who can assist with the implementation of the EHR or other types of informatics. "Superusers," often trained on the job, assist others to navigate the EHR. The clinical nurse specialist and nurse leader are patient care focused leadership roles.

12. How is a medication delivered by piggyback administered? A. It is pushed into the IV line. B. It is retrograded into the IV line. C. It is injected intramuscularly after another medication. D. It is mixed with 50 to 100 mL of IV fluid in a separate container.

D. It is mixed with 50 to 100 mL of IV fluid in a separate container. D. The medication is mixed in 50 to 100 mL of fluid in a separate container.

A nurse is planning to suction a patient's tracheostomy tube the day after its placement. Which action by the nurse is absolutely essential? A. Assessing the need to premedicate with an analgesic B. Placing the patient in low Fowler position C. Inserting the obturator into the outer cannula D. Maintaining aseptic technique

D. Maintaining aseptic technique d. Sterile technique is used for tracheal suctioning, to reduce the risk of introduction of disease-causing organisms. Aseptic technique is imperative to avoid introducing organisms into the lower airway. An obturator, which guides the direction of the outer cannula, is inserted into the tube during placement and removed once the outer cannula of the tube is in place. In the home setting, clean technique is used.

A nurse in a long-term care facility is on an interprofessional safety committee focusing on protecting older adults from injury and trauma. Which action does the nurse suggest they prioritize? A. Ensuring proper function of fire alarms B. Preventing exposure to temperature extremes C. Screening for partner or elder abuse D. Maintaining clutter free rooms and hallways

D. Maintaining clutter free rooms and hallways d. Falls among older adults are the most common cause of hospital admissions for trauma, therefore rooms and hallways should be free of clutter. Elder abuse, fires, and temperature extremes are also significant hazards for older adults but are not the most common cause of trauma admissions. IPV occurs more frequently in adults as opposed to older adults.

A nurse is testing a new computer program designed to store patient data. During which phase of testing does the nurse determine whether the system can handle high volumes of end users and care providers using the system at the same time? A. Unit B. Function C. Integration D. Performance

D. Performance d. Performance testing is technical and ensures proper functioning of the system and its ability to handle high volumes of care providers using the system at the same time. Unit testing is basic testing that occurs initially. Function testing uses test scripts to validate that a system is working as designed for one particular function. Integration testing uses test script to validate that a system is working as designed for a workflow that integrates multiple components of the system.

A patient who injured the spine in a motorcycle accident is receiving rehabilitation services in a short-term rehabilitation center. The nurse caring for the patient tells the AP not to place the patient in which position? A. Side-lying B. Fowler's C. Sims' D. Prone

D. Prone d. While placing the patient in the prone position for 30 minutes two or three times daily helps prevent knee and hip flexion contractures, it is contraindicated in patients who have spinal problems. The pull of gravity on the trunk when the patient lies prone produces a marked lordosis or forward curvature of the lumbar spine.

A nurse is filing a safety event report for a confused patient who fell while getting out of bed. Which action is most appropriate during documentation? A. Including suggestions on how to prevent the incident from recurring B. Providing minimal information about the incident C. Discussing the details with the patient before documenting them D. Recording the circumstances and effect on the patient in the health record

D. Recording the circumstances and effect on the patient in the health record d. A safety event report objectively describes the circumstances of the accident or incident. The report also details the patient's response and the examination and treatment of the patient after the incident. The nurse completes the event report immediately after the incident and is responsible for recording the circumstances and the effect on the patient in the medical record. The safety event report is not a part of the medical record and should not be mentioned in the documentation. Because laws vary in different states, nurses must know their own state law regarding safety event reports.

A pregnant woman has received an epidural analgesic prior to delivery. Assessment for which outcome to the medication will the nurse prioritize? A. Pruritus B. Urinary retention C. Vomiting D. Respiratory depression

D. Respiratory depression d. An opioid drug given by way of an epidural catheter or a displaced catheter may result in the occurrence of respiratory depression. Pruritus, urinary retention, and vomiting may occur but are not life threatening.

A patient with COPD is unable to perform personal hygiene without becoming exhausted. What nursing intervention would be appropriate for this patient? A. Assisting with all bathing and hygiene B. Telling the patient to avoid speaking during hygiene CC. Teaching the patient to take short shallow breaths during activity D. Taking rest periods between activities

D. Taking rest periods between activities d. To prevent fatigue during activities including hygiene, the nurse should group (personal care) activities into smaller steps and encourage rest periods between activities. The nurse promotes and maintains dignity, independence, and strength by assisting with activities when the patient has difficulty. The nurse should encourage the patient to voice feelings and concerns about self-care deficits and teach the patient to coordinate pursed-lip or diaphragmatic breathing with the activity.

A nurse is looking up information about the effects of a drug on different receptors. Characteristics of receptors include which of the following? A. They are carbohydrates located in cell membranes or inside cells. B. They are constantly synthesized and degraded in the body. C. They bind with molecules of any drug circulating in the bloodstream. D. They regulate the actions of all drugs.

D. They regulate the actions of all drugs. Receptors are mainly proteins that are manufactured and eliminated like other proteins in the body. Their numbers and binding capacities can be increased or decreased in particular situations. Most receptors bind with relatively few drugs. Some drugs do not require receptor binding for their actions (e.g., antacids such as TUMS or Mylanta).

Which of the following nursing actions will prevent adverse drug events? A. Use only the trade name when documenting medications. B. Crush long-acting medications if the patient has dysphagia. C. After receiving a verbal order, administer the medication and then write down the order. D. Use bar code technology according to institutional policy.

D. Use bar code technology according to institutional policy. D. Using bar code technology in the administration of medications will decrease medication administration errors.

A nurse is developing education for nurses and UAPs related to prevention of pressure injuries for residents in a long-term care facility. Which action to prevent pressure injury will the nurses delegate to the UAP? A. Maintaining the head of the bed elevated consistently B. Massaging over bony prominences C. Repositioning bedbound patients every 4 hours D. Using a mild cleansing agent when cleansing the skin

D. Using a mild cleansing agent when cleansing the skin d. To prevent pressure injuries, the nurse teaches the UAP to cleanse the skin routinely and whenever soiling occurs by using a mild cleansing agent with minimal friction, and avoiding hot water. The nurse educates the UAP to minimize the effects of shearing force by limiting the amount of time the head of the bed is elevated, when possible. Bony prominences should not be massaged, and bed-bound patients should be repositioned every 2 hours.

Determine the patient's risk for pressure injury using the Braden scale found in Figure 33-7, based on information in the electronic health record (EHR). EHR 1430 Admission Assessment S: Patient admitted from nursing home for sepsis, confusion, ambulatory dysfunction. B: 87-year-old patient, with history of heart failure and hypertension; comes to ED with shortness of breath and yellow sputum. A: Lungs with crackles, pale, short of breath on exertion, pulse oximetry 88%, skin fragile. Bedrest maintained. States has not eaten nor drank fluids for last 36 hours; incontinent of small amount of urine × 2. Responding to painful stimuli, not participating in turning or care. R: Need orders for oxygen, sputum culture, activity level. Consider IV fluids. J. Smith RN. A. No risk B. Moderate risk C. High risk D. Very high risk

D. Very high risk d. The patient is at very high risk for pressure injury. This patient responds only to painful stimulate (1); is occasionally moist (3); is bedridden (1); has not eaten (1), and requires maximum assistance for moving (1) for a total of 7 points. The Braden scale scoring is: a score of 19 to 23 indicates no risk; 15 to 18, mild risk; 13 to 14, moderate risk; 10 to 12, high risk; and 9 or lower, very high risk (Braden & Maklebust, 2005). In addition, nurses use clinical judgment to incorporate risk factors and/or other health problems into preventative interventions.

A nursing student attending clinical on a medical-surgical unit receives report from the off-going nurse stating the patient has adventitious breath sounds that clear after expectorating sputum. Which adventitious breath sound will the student expect to auscultate? A. Bronchial B. Bronchovesicular C. Vesicular D. Wheezing

D. Wheezing d. Wheezing and crackles represent adventitious or abnormal breath sounds. Bronchial, bronchovesicular, and vesicular breath sounds are normal.

A man is very upset with a drug recall of a medication he has been taking for a long time. He states that he feels like he can no longer trust anyone to protect him. In response to his questions about the process of drug development, it is important to know that with a new drug, the U.S. Food and Drug Administration (FDA) is responsible for A. testing the drug with animals B. testing the drug with healthy people C. marketing the drug to health care providers D. evaluating the drug for safety and effectiveness

D. evaluating the drug for safety and effectiveness The U.S. Food and Drug Administration reviews studies reported by others, mainly pharmaceutical companies, and determines whether a drug is sufficiently safe and effective to be marketed. The drug manufacturer is responsible for testing the drug (in animals and people) and marketing the drug.

A nurse practitioner (NP) has just changed a patient's medication from an oral form to a patch formulation to avoid the first-pass effect. The NP has explained it to the patient, but the patient still has questions and asks the nurse to explain again what is meant by the first-pass effect. The nurse would be most correct in explaining that this has to do with how A. drugs initially bind to plasma proteins B. initial renal function is involved in drug excretion C. the way drugs first reach their target cells D. initial metabolism of an oral drug occurs before it reaches the systemic circulation

D. initial metabolism of an oral drug occurs before it reaches the systemic circulation The "first-pass effect" involves metabolism of an oral drug in the liver so that only a portion of a drug dose reaches the systemic circulation and becomes available to act on target tissues or to be eliminated from the body. Drug binding to plasma proteins affects drug distribution; an increased amount of drug has to be absorbed before effective therapeutic levels of unbound drug are reached.

In understanding the use of controlled substances for patients, it is important that the nurse knows that controlled drugs are A. categorized according to prescription or nonprescription status B. regulated by state and local laws more than federal laws C. those that must demonstrate high standards of safety D. scheduled according to medical use and potential for abuse

D. scheduled according to medical use and potential for abuse Controlled or scheduled drugs are categorized by federal law according to use and abuse potential. These drugs have regulations that govern prescribing, dispensing, administering, and record-keeping. Almost all scheduled drugs require prescriptions (i.e., cannot legally be sold over the counter). Basic laws are federal, and states and local communities may enact additional laws.

Two nurses are repositioning a patient and pulling the patient up in bed. Which of these steps is most appropriate to prevent injury to the nurses? A. Telling the patient to cross their arms and legs B. Pulling the patient from underneath the axilla toward the top of the bed C. Avoiding using a draw sheet to lift or reposition the patient D. Ensuring the bed is at the level of the nurses' hips E. Facing the head of the bed and rocking in synchrony

The nurses should face the direction the patient will move and rock in synchrony prior to moving the patient in that direction. A lifting or repositioning sheet or device is used to decrease friction and facilitate movement. While the patient can cross their arms, they can also be instructed to press their feet into the mattress to assist movement. The bed should be at the level of the nurses' elbows.

A patient is to receive an intramuscular injection of ketorolac. Which of the following muscles should be avoided? A. deltoid B. dorsogluteal C. ventrogluteal D. vastus lateralis

idkkk

Differences in CYP-450 drug-metabolizing enzymes are known to cause genetic variation in the drug metabolism of certain drugs that increase the risk of adverse effects. These include which of the following? (Select all that apply.) A. CYP2D6 metabolism of several antidepressant, antipsychotic, and beta-adrenergic blocker drugs that increases the risk of drug accumulation and adverse effects. B. CYP2D6 metabolism of codeine in individuals who are ultrarapid metabolizers. The conversion of codeine to morphine occurs quickly and poses a risk of serious adverse effects, such as respiratory depression. C. CYP2C19 metabolism in some individuals of Asian descent. This may cause decreased drug metabolism of diazepam, omeprazole, and some antidepressants leading to adverse effects. D. Individuals with a deficiency of glucose-6-phosphate dehydrogenase, who may have hemolytic anemia when given antimalarial drugs, sulfonamides, analgesics, antipyretics, and other drugs.

no answer in book found in Frand/Penn chapter 2 question 7 pg 33 investigate further


Kaugnay na mga set ng pag-aaral

FIN 320 chap 4 Time Value of Money: Valuing Cash Flow Streams

View Set

Bontrager Chapter 4 - Forearm and elbow

View Set

Assessment of the Cardiovascular System

View Set

Mental Health, Emotional Health, and Stress-Related Problems

View Set

London - the capital of the UK - tasks

View Set